You are on page 1of 61

Nat Res Cases

G.R. No. 164408 March 24, 2014

REPUBLIC OF THE PHILIPPINES, Petitioner,


vs.
ZURBARAN REALTY AND DEVELOPMENT CORPORATION, Respondent.

DECISION

BERSAMIN, J.:

An application for original registration of land of the public domain under Section 14(2) of Presidential Decree (PD) No. 1529
must show not only that the land has previously been declared alienable and disposable, but also that the land has been
declared patrimonial property of the State at the onset of the 30-year or 10-year period of possession and occupation required
under the law on acquisitive prescription. Once again, the Court applies this rule-as clarified in Heirs of Mario Malabanan v.
Republic1 – in reviewing the decision promulgated on June 10, 2004,2 whereby the Court of Appeals (CA) granted the
petitioner's application for registration of land.

Antecedents

On May 28, 1993, respondent Zurbaran Realty and Development Corporation filed in the Regional Trial Court (RTC) in San
Pedro, Laguna an application for original registration covering a 1,520 square meter parcel of land situated in Barrio Banlic,
Municipality of Cabuyao, Province of Laguna, denominated as Lot 8017-A of Subdivision Plan CSD-04-006985-D, Cad. 455-D,
Cabuyao Cadastre,3 alleging that it had purchased the land on March 9, 1992 from Jane de Castro Abalos, married to Jose
Abalos, for P300,000.00; that the land was declared for taxation purposes in the name of its predecessor-in-interest under Tax
Declaration No. 22711; that there was no mortgage or encumbrance of any kind affecting the land, nor was there any other
person or entity having any interest thereon, legal or equitable, adverse to that of the applicant; and that the applicant and its
predecessors-in-interest had been in open, continuous and exclusive possession and occupation of the land in the concept of an
owner.

Attached to the application were several documents, namely: (1) tracing cloth plan as approved by the Land Management
Division of the Department of Environment and Natural Resources (DENR); (2) blue print copies of the tracing cloth plan; (3)
copies of the technical description; (4) copies of Tax Declaration No. 2711; and (5) copies of the Deed of Sale dated March 9,
1992.

The Republic, represented by the Director of Lands, opposed the application, arguing that the applicant and its predecessors-in-
interest had not been in open, continuous, exclusive and notorious possession and occupation of the land since June 12, 1945;
that the muniments of title and tax declaration presented did not constitute competent and sufficient evidence of a bona fide
acquisition of the land; and that the land was a portion of the public domain, and, therefore, was not subject to private
appropriation.4

The RTC directed the Land Management Bureau, Manila; the Community Environment and Natural Resources Office (CENRO) of
Los Baños, Laguna; and the Land Management Sector and Forest Management Bureau, Manila, to submit a status report on the
land, particularly, on whether the land was covered by a land patent, whether it was subject of a previously approved isolated
survey, and whether it was within a forest zone.5

In his memorandum to the DENR, Region IV (Lands Forestry Sector), and the Provincial Prosecutor of Laguna, a copy of which
was furnished the trial court, CENRO Officer Arnulfo Hernandez stated that the land had been "verified to be within the
Alienable and Disposable land under Land Classification Project No. 23-A of Cabuyao, Laguna, certified and declared as such
pursuant to the provisions of Presidential Decree No. 705, as amended, under Forestry Administrative Order No. A-1627 dated
September 28, 1981 per BFD Map LC-3004." Attached to the memorandum was the inspection report declaring that "the area is
surrounded with concrete fence, three (3) buildings for employees’ residence;" that the land was acquired through sale before
the filing of the application; that the applicant and its predecessors-in-interest had been in "continuous, open and peaceful
occupation" of the land, and that "no forestry interest is adversely affected."6

CENRO Land Management Inspector/Investigator Rodolfo S. Gonzales reported that: (1) the land was covered by a survey plan
approved by the Regional Land Director/Land Registration Authority on May 25, 1988 pursuant to PD No. 239 dated July 9,
1975; (2) it consisted of 22,773 square meters and was located in Barangay Banlic, Cabuyao, Laguna; (3) the area was entirely
within the alienable and disposable area; (4) it had never been forfeited in favor of the government for non-payment of taxes,
and had not been confiscated in connection with any civil or criminal cases; (5) it was not within a previously patented property
as certified to by the Register of Deeds, Calamba, Laguna; and (6) there was no public land application filed for it by the
applicant or any other persons as per verification from the records unit of his office. The report further stated that a verification
at the Office of the Municipal Assessor showed that: (1) the land was declared for the first time in 1960 under Tax Declaration
No. 6712 in the name of Enrique Hemedez with an area of 23,073 square meters; (2) it was now covered by Tax Declaration No.
2253 issued in the name of the respondent; (3) the real property taxes had been paid since 1968; and (4) it had not been
earmarked for public or quasi-public purposes per information from the District Engineer.

After inspection, it was also found that (1) the land was residential; (2) the respondent was in the actual occupation and
possession of the land; and (3) the land did not encroach upon an established watershed, riverbank/bed protection, creek,
right-of-way or park site or any area devoted to general use or devoted to public service.7

A certification was issued by the Records Management Division of the Land Management Bureau stating that it had no record
of any kind of public land applications/land patents covering the parcel of land subject of the application. 8

The respondent presented Gloria P. Noel, its Vice President and Treasurer, who testified that the respondent had purchased
the land from Jane de Castro Abalos on March 9, 1992 for P300,000.00; that the land had been declared for taxation purposes
in the name of Abalos under Tax Declaration No. 22711; that after the sale, a new Tax Declaration had been issued in the name
of the respondent, who had meanwhile taken possession of the land by building a fence around it and introducing
improvements thereon; that the respondent had paid the real property taxes thereon since its acquisition; that the
respondent’s possession had been continuous, open and public; and that the land was free from any lien or encumbrance; and
that there was no adverse claimant to the land.9

Engr. Edilberto Tamis attested that he was familiar with the land because it was a portion of Lot No. 8017 of Subdivision Plan
Cad-455-D of the Cabuyao Cadastre, owned by Corazon Tapalla who had acquired it from the Hemedez family; that Tapalla had
sold a portion of Lot No. 8017 to Abalos and the remaining portion to him; and that he had witnessed the sale of the land to the
respondent.10

The respondent’s final witness was Armando Espela who declared that he was a retired land overseer residing in Barangay
Banlic from birth; that he was familiar with the land which was part of a bigger parcel of land owned by the Hemedez family;
that his father, Toribio Espela, with his assistance, and one Francisco Capacio worked on the land since 1960; that the entire
landholding had originally been sugarland, but was later on subdivided, sold, and resold until it ceased to be agricultural land;
that, in 1982, the land was sold to Corazon Tapalla who hired him as the overseer; that as the overseer, he fenced and cleared
the area; that he was allowed to use the grassy portion for grazing purposes; that in 1987, Tapalla sold part of the land to
Abalos and the remaining portion to Engr. Tamis; that he continued to oversee the land for the new owners; that Abalos then
sold her portion to the respondent in 1992; that since then, the respondent took possession of the land, and he then ceased to
be the overseer; that the possession by the Hemedez family and its successors-in-interest was open, continuous, public and
under claim of ownership; and that he did not know any person who claimed ownership of the land other than those he and his
father served as overseers.11

Decision of the RTC

On May 12, 1997, the RTC rendered its decision, holding that the respondent and its predecessors-in-interest had been in open,
public, peaceful, continuous, exclusive and adverse possession and occupation of the land under a bona fide claim of ownership
even prior to 1960 and, accordingly, granted the application for registration, viz:
WHEREFORE, taking into consideration the evidence submitted by the applicant, this Court hereby orders the confirmation and
registration of title of the land described as Lot 8017-A of subdivision plan Csd-04-006985-D, being a portion of Lot 8017 of
subdivision plan Cad-455-D, Cabuyao Cadastre situated at Barangay Banlic, Cabuyao, Laguna with an area of 1,520 square
meters to be entered under the name of the applicant Zurbaran Realty and Development Corporation, a corporation organized
and existing under the laws of the Philippines with office address at 33 M. Viola St., San Francisco del Monte, Quezon City by
the Land Registration Authority. After the decision shall become final, let an order for the issuance of a decree of title be issued
in favor of said applicant.

SO ORDERED.12

Judgment of the CA

The Republic appealed, arguing that the issue of whether the applicant and its predecessors-in-interest had possessed the land
within the required length of time could not be determined because there was no evidence as to when the land had been
declared alienable and disposable.

On June 10, 2004, the CA promulgated its judgment affirming the RTC, and concluded that the reports made by the concerned
government agencies and the testimonies of those familiar with the land in question had buttressed the court a quo’s
conclusion that the respondent and its predecessors-in-interest had been in open, public, peaceful, continuous, exclusive, and
adverse possession and occupation of the land under a bona fide claim of ownership even prior to 1960. 13

Issue

Hence, the Republic appeals the adverse judgment of the CA upon the following ground:

THE COURT OF APPEALS GRAVELY ERRED ON A QUESTION OF LAW WHEN IT AFFIRMED THE TRIAL COURT’S GRANT OF THE
APPLICATION FOR ORIGINAL REGISTRATION DESPITE THE ABSENCE OF EVIDENCE THAT RESPONDENT AND ITS PREDECESSORS-
IN-INTEREST HAVE COMPLIED WITH THE PERIOD OF POSSESSION AND OCCUPATION REQUIRED BY LAW. 14

The Republic contends that the respondent did not establish the time when the land covered by the application for registration
became alienable and disposable;15 that such detail was crucial because the possession of the respondent and its predecessors-
in-interest, for the purpose of determining whether it acquired the property by prescription, should be reckoned from the time
when the land was declared alienable and disposable; and that prior to the declaration of the land of the public domain as
alienable and disposable, it was not susceptible to private ownership, and any possession or occupation at such time could not
be counted as part of the period of possession required under the law on prescription.16

The respondent counters that whether it established when the property was declared alienable and disposable and whether it
complied with the 30-year required period of possession should not be entertained anymore by the Court because: (a) these
issues had not been raised in the trial court and were being raised for the first time on appeal; and (b) factual findings of the
trial court, especially when affirmed by the CA, were binding and conclusive on this Court. At any rate, the respondent insists
that it had been in open, public, peaceful, continuous, and adverse possession of the property for the prescribed period of 30
years as evidenced by the fact that the property had been declared for taxation purposes in 1960 in the name of its
predecessors-in-interest, and that such possession had the effect of converting the land into private property and vesting
ownership upon the respondent.17

In reply, the Republic asserts that it duly opposed the respondent’s application for registration; that it was only able to
ascertain the errors committed by the trial court after the latter rendered its decision; and that the burden of proof in land
registration cases rested on the applicant who must prove its ownership of the property being registered. The Republic
maintains that the Court had the authority to review and reverse the factual findings of the lower courts when the conclusion
reached was not supported by the evidence on record, as in this case.18

Ruling
The petition for review is meritorious.

Section 14 of P.D. No. 1529 enumerates those who may file an application for registration of land based on possession and
occupation of a land of the public domain, thus:

Section 14. Who may apply. The following persons may file in the proper Court of First Instance an application for registration
of title to land, whether personally or through their duly authorized representatives:

(1) Those who by themselves or through their predecessors-in-interest have been in open, continuous, exclusive and notorious
possession and occupation of alienable and disposable lands of the public domain under a bona fide claim of ownership since
June 12, 1945, or earlier.

(2) Those who have acquired ownership of private lands by prescription under the provision of existing laws.

xxxx

An application for registration under Section14(1) of P.D. No. 1529 must establish the following requisites, namely: (a) the land
is alienable and disposable property of the public domain; (b) the applicant and its predecessors in interest have been in open,
continuous, exclusive and notorious possession and occupation of the land under a bona fide claim of ownership; and (c) the
applicant and its predecessors-in-interest have possessed and occupied the land since June 12, 1945, or earlier. The Court has
clarified in Malabanan19 that under Section14(1), it is not necessary that the land must have been declared alienable and
disposable as of June 12, 1945, or earlier, because the law simply requires the property sought to be registered to be alienable
and disposable at the time the application for registration of title is filed. The Court has explained that a contrary interpretation
would absurdly limit the application of the provision "to the point of virtual inutility."

The foregoing interpretation highlights the distinction between a registration proceeding filed under Section 14(1) of P.D. No.
1529 and one filed under Section 14(2) of P.D. No. 1529. According to Malabanan:

Section 14(1) mandates registration on the basis of possession, while Section 14(2) entitles registration on the basis of
prescription. Registration under Section 14(1) is extended under the aegis of the Property Registration Decree and the Public
Land Act while registration under Section 14(2) is made available both by the Property Registration Decree and the Civil Code.20

In other words, registration under Section 14(1) of P.D. No. 1529 is based on possession and occupation of the alienable and
disposable land of the public domain since June 12, 1945 or earlier, without regard to whether the land was susceptible to
private ownership at that time. The applicant needs only to show that the land had already been declared alienable and
disposable at any time prior to the filing of the application for registration.

On the other hand, an application under Section 14(2) of P.D. No. 1529 is based on acquisitive prescription and must comply
with the law on prescription as provided by the Civil Code. In that regard, only the patrimonial property of the State may be
acquired by prescription pursuant to the Civil Code.21 For acquisitive prescription to set in, therefore, the land being possessed
and occupied must already be classified or declared as patrimonial property of the State. Otherwise, no length of possession
would vest any right in the possessor if the property has remained land of the public dominion. Malabanan stresses that even if
the land is later converted to patrimonial property of the State, possession of it prior to such conversion will not be counted to
meet the requisites of acquisitive prescription.22 Thus, registration under Section 14(2) of P.D. No. 1529 requires that the land
had already been converted to patrimonial property of the State at the onset of the period of possession required by the law on
prescription.

An application for registration based on Section 14(2) of P.D. No. 1529 must, therefore, establish the following requisites, to
wit: (a) the land is an alienable and disposable, and patrimonial property of the public domain; (b) the applicant and its
predecessors-in-interest have been in possession of the land for at least 10 years, in good faith and with just title, or for at least
30 years, regardless of good faith or just title; and (c) the land had already been converted to or declared as patrimonial
property of the State at the beginning of the said 10-year or 30-year period of possession.
To properly appreciate the respondent’s case, we must ascertain under what provision its application for registration was filed.
If the application was filed under Section 14(1) of P.D. No. 1529, the determination of the particular date when the property
was declared alienable and disposable would be unnecessary, inasmuch as proof showing that the land had already been
classified as such at the time the application was filed would be enough. If the application was filed under Section 14(2) of P.D.
No. 1529, the determination of the issue would not be crucial for, as earlier clarified, it was not the declaration of the land as
alienable and disposable that would make it susceptible to private ownership by acquisitive prescription. Malabanan expounds
thereon, thus …Would such lands so declared alienable and disposable be converted, under the Civil Code, from property of the
public dominion into patrimonial property? After all, by connotative definition, alienable and disposable lands may be the
object of the commerce of man; Article 1113 provides that all things within the commerce of man are susceptible to
prescription; and the same provision further provides that patrimonial property of the State may be acquired by prescription.

Nonetheless, Article 422 of the Civil Code states that "[p]roperty of public dominion, when no longer intended for public use or
for public service, shall form part of the patrimonial property of the State." It is this provision that controls how public dominion
property may be converted into patrimonial property susceptible to acquisition by prescription. After all, Article 420 (2) makes
clear that those property "which belong to the State, without being for public use, and are intended for some public service or
for the development of the national wealth" are public dominion property. For as long as the property belongs to the State,
although already classified as alienable or disposable, it remains property of the public dominion if when it is "intended for
some public service or for the development of the national wealth."

Accordingly, there must be an express declaration by the State that the public dominion property is no longer intended for
public service or the development of the national wealth or that the property has been converted into patrimonial. Without
such express declaration, the property, even if classified as alienable or disposable, remains property of the public dominion,
pursuant to Article 420(2), and thus incapable of acquisition by prescription. It is only when such alienable and disposable lands
are expressly declared by the State to be no longer intended for public service or for the development of the national wealth
that the period of acquisitive prescription can begin to run. Such declaration shall be in the form of a law duly enacted by
Congress or a Presidential Proclamation in cases where the President is duly authorized by law.23

The respondent’s application does not enlighten as to whether it was filed under Section 14(1) or Section 14(2) of P.D. No.
1529. The application alleged that the respondent and its predecessors-in-interest had been in open, continuous and exclusive
possession and occupation of the property in the concept of an owner, but did not state when possession and occupation
commenced and the duration of such possession. At any rate, the evidence presented by the respondent and its averments in
the other pleadings reveal that the application for registration was filed based on Section 14(2), not Section 14(1) of P.D. No.
1529. The respondent did not make any allegation in its application that it had been in possession of the property since June 12,
1945, or earlier, nor did it present any evidence to establish such fact.1âwphi1

With the application of the respondent having been filed under Section 14(2) of P.D. No. 1529, the crucial query is whether the
land subject of the application had already been converted to patrimonial property of the State. In short, has the land been
declared by law as no longer intended for public service or the development of the national wealth?

The respondent may perhaps object to a determination of this issue by the Court for the same reason that it objects to the
determination of whether it established when the land was declared alienable and disposable, that is, the issue was not raised
in and resolved and by the trial court. But the objection would be futile because the issue was actually raised in the trial court,
as borne out by the Republic's allegation in its opposition to the application to the effect "that the land is a portion of the public
domain not subject to prescription." In any case, the interest of justice dictates the consideration and resolution of an issue that
is relevant to another that was specifically raised. The rule that only theories raised in the initial proceedings may be taken up
by a party on appeal refers only to independent, not concomitant, matters to support or oppose the cause of action. 24

Here, there is no evidence showing that the land in question was within an area expressly declared by law either to be the
patrimonial property of the State, or to be no longer intended for public service or the development of the national wealth. The
Court is left with no alternative but to deny the respondent's application for registration.
WHEREFORE, the Court GRANTS the petition for review on certiorari; REVERSES and SETS ASIDE the decision promulgated on
June 10, 2004; and DISMISSES the respondent's application for original registration of Lot 8017-A of Subdivision Plan CSD-04-
006985-D, Cad. 455-D, of the Cabuyao Cadastre.

No pronouncement on costs of suit.

SO ORDERED.

LUCAS P. BERSAMIN
Associate Justice

G.R. No. 186639 February 5, 2014

REPUBLIC OF THE PHILIPPINES, Petitioner,


vs.
EMMANUEL C. CORTEZ, Respondent.

DECISION

REYES, J.:

Before this Court is a petition for review on certiorari1 under Rule 45 of the Rules of Court seeking to annul and set aside the
Decision2 dated February 17, 2009 of the Court of Appeals (CA) in CA-G.R. CV No. 87505. The CA affirmed the Decision3 dated
February 7, 2006 of the Regional Trial Court (RTC) of Pasig City, Branch 68, in LRC Case No. N-11496.

The Facts

On February 28, 2003, respondent Emmanuel C. Cortez (Cortez) filed with the RTC an application4 for judicial confirmation of
title over a parcel of land located at Barangay (Poblacion) Aguho, P. Herrera Street, Pateros, Metro Manila. The said parcel of
land has an area of 110 square meters and more particularly described as Lot No. 2697-B of the Pateros Cadastre. In support of
his application, Cortez submitted, inter alia, the following documents: (1) tax declarations for various years from 1966 until
2005; (2) survey plan of the property, with the annotation that the property is classified as alienable and disposable; (3)
technical description of the property, with a certification issued by a geodetic engineer; (4) tax clearance certificate; (5)
extrajudicial settlement of estate dated March 21, 1998, conveying the subject property to Cortez; and (6) escritura de particion
extrajudicial dated July 19, 1946, allocating the subject property to Felicisima Cotas – Cortez’ mother.

As there was no opposition, the RTC issued an Order of General Default and Cortez was allowed to present his evidence ex-
parte.

Cortez claimed that the subject parcel of land is a portion of Lot No. 2697, which was declared for taxation purposes in the
name of his mother. He alleged that Lot No. 2697 was inherited by his mother from her parents in 1946; that, on March 21,
1998, after his parents died, he and his siblings executed an Extra-Judicial Settlement of Estate over the properties of their
deceased parents and one of the properties allocated to him was the subject property. He alleged that the subject property had
been in the possession of his family since time immemorial; that the subject parcel of land is not part of the reservation of the
Department of Environment and Natural Resources (DENR) and is, in fact, classified as alienable and disposable by the Bureau
of Forest Development (BFD).

Cortez likewise adduced in evidence the testimony of Ernesto Santos, who testified that he has known the family of Cortez for
over sixty (60) years and that Cortez and his predecessors-in-interest have been in possession of the subject property since he
came to know them.

On February 7, 2006, the RTC rendered a Decision,5 which granted Cortez’ application for registration, viz:
WHEREFORE, finding the application meritorious, the Court DECLARES, CONFIRMS, and ORDERS the registration of the
applicant’s title thereto.

As soon as this Decision shall have become final and after payment of the required fees, let the corresponding Decrees be
issued in the name of the applicant, Emmanuel C. Cortez.

Let copies of this Decision be furnished the Office of the Solicitor General, Land Registration Authority, Land Management
Bureau, and the Registry of Deeds of Rizal.

SO ORDERED.6

In granting Cortez’ application for registration of title to the subject property, the RTC made the following ratiocinations:

From the foregoing, the Court finds that there is sufficient basis to grant the relief prayed for. It having been established by
competent evidence that the possession of the land being applied for by the applicant and his predecessor-in-interest have
been in open, actual, uninterrupted, and adverse possession, under claim of title and in the concept of owners, all within the
time prescribed by law, the title of the applicant should be and must be AFFIRMED and CONFIRMED.7

The Republic of the Philippines (petitioner), represented by the Office of the Solicitor General, appealed to the CA, alleging that
the RTC erred in granting the application for registration despite the failure of Cortez to comply with the requirements for
original registration of title. The petitioner pointed out that, although Cortez declared that he and his predecessors-in-interest
were in possession of the subject parcel of land since time immemorial, no document was ever presented that would establish
his predecessors-in-interest’s possession of the same during the period required by law. That petitioner claimed that Cortez’
assertion that he and his predecessors-in-interest had been in open, adverse, and continuous possession of the subject
property for more than thirty (30) years does not constitute well-neigh incontrovertible evidence required in land registration
cases; that it is a mere claim, which should not have been given weight by the RTC.

Further, the petitioner alleged that there was no certification from any government agency that the subject property had
already been declared alienable and disposable. As such, the petitioner claims, Cortez’ possession of the subject property, no
matter how long, cannot confer ownership or possessory rights.

On February 17, 2009, the CA, by way of the assailed Decision,8 dismissed the petitioner’s appeal and affirmed the RTC Decision
dated February 7, 2006. The CA ruled that Cortez was able to prove that the subject property was indeed alienable and
disposable, as evidenced by the declaration/notation from the BFD.

Further, the CA found that Cortez and his predecessors-in-interest had been in open, continuous, and exclusive possession of
the subject property for more than 30 years, which, under Section 14(2) of Presidential Decree (P.D.) No. 1529 9, sufficed to
convert it to private property. Thus:

It has been settled that properties classified as alienable and disposable land may be converted into private property by reason
of open, continuous and exclusive possession of at least 30 years. Such property now falls within the contemplation of "private
lands" under Section 14(2) of PD 1529, over which title by prescription can be acquired. Thus, under the second paragraph of
Section 14 of PD 1529, those who are in possession of alienable and disposable land, and whose possession has been
characterized as open, continuous and exclusive for 30 years or more, may have the right to register their title to such land
despite the fact that their possession of the land commenced only after 12 June 1945. x x x

xxxx

While it is significant to note that applicant-appellee’s possession of the subject property can be traced from his mother’s
possession of the same, the records, indeed, show that his possession of the subject property, following Section 14(2) [of PD
1529], is to be reckoned from January 3, 1968, when the subject property was declared alienable and disposable and not way
back in 1946, the year when he inherited the same from his mother. At any rate, at the time the application for registration was
filed in 2003, there was already sufficient compliance with the requirement of possession. His possession of the subject
property has been characterized as open, continuous, exclusive and notorious possession and occupation in the concept of an
owner.10 (Citations omitted)

Hence, the instant petition.

The Issue

The sole issue to be resolved by the Court is whether the CA erred in affirming the RTC Decision dated February 7, 2006, which
granted the application for registration filed by Cortez.

The Court’s Ruling

The petition is meritorious.

At the outset, the Court notes that the RTC did not cite any specific provision of law under which authority Cortez’ application
for registration of title to the subject property was granted. In granting the application for registration, the RTC merely stated
that "the possession of the land being applied for by [Cortez] and his predecessor-in-interest have been in open, actual,
uninterrupted, and adverse possession, under claim of title and in the concept of owners, all within the time prescribed by
law[.]"11 On the other hand, the CA assumed that Cortez’ application for registration was based on Section 14(2) of P.D. No.
1529. Nevertheless, Cortez, in the application for registration he filed with the RTC, proffered that should the subject property
not be registrable under Section 14(2) of P.D. No. 1529, it could still be registered under Section 48(b) of Commonwealth Act
No. 141 (C.A. No. 141), or the Public Land Act, as amended by P.D. No. 107312 in relation to Section 14(1) of P.D. No. 1529. Thus,
the Court deems it proper to discuss Cortez’ application for registration of title to the subject property vis-à-vis the provisions of
Section 14(1) and (2) of P.D. No. 1529.

Applicants for original registration of title to land must establish compliance with the provisions of Section 14 of P.D. No. 1529,
which pertinently provides that:

Sec. 14. Who may apply. The following persons may file in the proper Court of First Instance an application for registration of
title to land, whether personally or through their duly authorized representatives:

(1) Those who by themselves or through their predecessors-in interest have been in open, continuous, exclusive and notorious
possession and occupation of alienable and disposable lands of the public domain under a bona fide claim of ownership since
June 12, 1945, or earlier.

(2) Those who have acquired ownership of private lands by prescription under the provision of existing laws.

xxxx

After a careful scrutiny of the records of this case, the Court finds that Cortez failed to comply with the legal requirements for
the registration of the subject property under Section 14(1) and (2) of P.D. No. 1529.

Section 14(1) of P.D. No. 1529 refers to the judicial confirmation of imperfect or incomplete titles to public land acquired under
Section 48(b) of C.A. No. 141, as amended by P.D. No. 1073. "Under Section 14(1) [of P.D. No. 1529], applicants for registration
of title must sufficiently establish first, that the subject land forms part of the disposable and alienable lands of the public
domain; second, that the applicant and his predecessors-in-interest have been in open, continuous, exclusive, and notorious
possession and occupation of the same; and third, that it is under a bona fide claim of ownership since June 12, 1945, or
earlier."13

The first requirement was not satisfied in this case. To prove that the subject property forms part of the alienable and
disposable lands of the public domain, Cortez adduced in evidence a survey plan Csd-00-00063314(conversion-subdivision plan
of Lot 2697, MCadm 594-D, Pateros Cadastral Mapping) prepared by Geodetic Engineer Oscar B. Fernandez and certified by the
Lands Management Bureau of the DENR. The said survey plan contained the following annotation:
This survey is inside L.C. Map No. 2623, Project No. 29, classified as alienable & disposable by the Bureau of Forest
Development on Jan. 3, 1968.

However, Cortez’ reliance on the foregoing annotation in the survey plan is amiss; it does not constitute incontrovertible
evidence to overcome the presumption that the subject property remains part of the inalienable public domain. In Republic of
the Philippines v. Tri-Plus Corporation,15 the Court clarified that, the applicant must at the very least submit a certification from
the proper government agency stating that the parcel of land subject of the application for registration is indeed alienable and
disposable, viz:

It must be stressed that incontrovertible evidence must be presented to establish that the land subject of the application is
alienable or disposable.

In the present case, the only evidence to prove the character of the subject lands as required by law is the notation appearing
in the Advance Plan stating in effect that the said properties are alienable and disposable. However, this is hardly the kind of
proof required by law. To prove that the land subject of an application for registration is alienable, an applicant must establish
the existence of a positive act of the government such as a presidential proclamation or an executive order, an administrative
action, investigation reports of Bureau of Lands investigators, and a legislative act or statute. The applicant may also secure a
certification from the Government that the lands applied for are alienable and disposable. In the case at bar, while the Advance
Plan bearing the notation was certified by the Lands Management Services of the DENR, the certification refers only to the
technical correctness of the survey plotted in the said plan and has nothing to do whatsoever with the nature and character of
the property surveyed. Respondents failed to submit a certification from the proper government agency to prove that the lands
subject for registration are indeed alienable and disposable.16 (Citations omitted and emphasis ours)

Similarly, in Republic v. Roche,17 the Court declared that:

Respecting the third requirement, the applicant bears the burden of proving the status of the land. In this connection, the Court
has held that he must present a certificate of land classification status issued by the Community Environment and Natural
Resources Office (CENRO) or the Provincial Environment and Natural Resources Office (PENRO) of the DENR. He must also
prove that the DENR Secretary had approved the land classification and released the land as alienable and disposable, and that
it is within the approved area per verification through survey by the CENRO or PENRO. Further, the applicant must present a
copy of the original classification approved by the DENR Secretary and certified as true copy by the legal custodian of the official
records. These facts must be established by the applicant to prove that the land is alienable and disposable.

Here, Roche did not present evidence that the land she applied for has been classified as alienable or disposable land of the
public domain. She submitted only the survey map and technical description of the land which bears no information regarding
the land’s classification. She did not bother to establish the status of the land by any certification from the appropriate
government agency. Thus, it cannot be said that she complied with all requisites for registration of title under Section 14(1) of
P.D. 1529.18 (Citations omitted and emphasis ours)

The annotation in the survey plan presented by Cortez is not the kind of evidence required by law as proof that the subject
property forms part of the alienable and disposable land of the public domain. Cortez failed to present a certification from the
proper government agency as to the classification of the subject property. Cortez likewise failed to present any evidence
showing that the DENR Secretary had indeed classified the subject property as alienable and disposable. Having failed to
present any incontrovertible evidence, Cortez’ claim that the subject property forms part of the alienable and disposable lands
of the public domain must fail.

Anent the second and third requirements, the Court finds that Cortez likewise failed to establish the same.1âwphi1 Cortez
failed to present any evidence to prove that he and his predecessors-in-interest have been in open, continuous, exclusive, and
notorious possession and occupation of the subject property since June 12, 1945, or earlier. Cortez was only able to present
oral and documentary evidence of his and his mother’s ownership and possession of the subject property since 1946, the year
in which his mother supposedly inherited the same.
Other than his bare claim that his family possessed the subject property since time immemorial, Cortez failed to present any
evidence to show that he and his predecessors-in-interest indeed possessed the subject property prior to 1946; it is a mere
claim and not factual proof of possession. "It is a rule that general statements that are mere conclusions of law and not factual
proof of possession are unavailing and cannot suffice. An applicant in a land registration case cannot just harp on mere
conclusions of law to embellish the application but must impress thereto the facts and circumstances evidencing the alleged
ownership and possession of the land."19

Further, the earliest tax declaration presented by Cortez was only in 1966. Cortez failed to explain why, despite his claim that he
and his predecessors-in-interest have been in possession of the subject property since time immemorial, it was only in 1966
that his predecessors-in-interest started to declare the same for purposes of taxation.

That Cortez and his predecessors-in-interest have been in possession of the subject property for fifty-seven (57) years at the
time he filed his application for registration in 2003 would likewise not entitle him to registration thereof under Section 14(2) of
P.D. No. 1529.

Section 14(2) of P.D. No. 1529 sanctions the original registration of lands acquired by prescription under the provisions of
existing laws. "As Section 14(2) [of P.D. No. 1529] categorically provides, only private properties may be acquired thru
prescription and under Articles 420 and 421 of the Civil Code, only those properties, which are not for public use, public service
or intended for the development of national wealth, are considered private." 20

In Heirs of Mario Malabanan v. Republic,21 the Court however clarified that lands of the public domain that are patrimonial in
character are susceptible to acquisitive prescription and, accordingly, eligible for registration under Section 14(2) of P.D. No.
1529, viz:

The Civil Code makes it clear that patrimonial property of the State may be acquired by private persons through prescription.
This is brought about by Article 1113, which states that "[a]ll things which are within the commerce of man are susceptible to
prescription," and that property of the State or any of its subdivisions not patrimonial in character shall not be the object of
prescription."

There are two modes of prescription through which immovables may be acquired under the Civil Code.1âwphi1 The first is
ordinary acquisitive prescription, which, under Article 1117, requires possession in good faith and with just title; and, under
Article 1134, is completed through possession of ten (10) years. There is nothing in the Civil Code that bars a person from
acquiring patrimonial property of the State through ordinary acquisitive prescription, nor is there any apparent reason to
impose such a rule. At the same time, there are indispensable requisites–good faith and just title. The ascertainment of good
faith involves the application of Articles 526, 527, and 528, as well as Article 1127 of the Civil Code, provisions that more or less
speak for themselves.22 (Citation omitted and emphasis ours)

The Court nevertheless emphasized that there must be an official declaration by the State that the public dominion property is
no longer intended for public use, public service, or for the development of national wealth before it can be acquired by
prescription; that a mere declaration by government officials that a land of the public domain is already alienable and
disposable would not suffice for purposes of registration under Section 14(2) of P.D. No. 1529. The Court further stressed that
the period of acquisitive prescription would only begin to run from the time that the State officially declares that the public
dominion property is no longer intended for public use, public service, or for the development of national wealth. Thus:

Let us now explore the effects under the Civil Code of a declaration by the President or any duly authorized government officer
of alienability and disposability of lands of the public domain. Would such lands so declared alienable and disposable be
converted, under the Civil Code, from property of the public dominion into patrimonial property? After all, by connotative
definition, alienable and disposable lands may be the object of the commerce of man; Article 1113 provides that all things
within the commerce of man are susceptible to prescription; and the same provision further provides that patrimonial property
of the State may be acquired by prescription.

Nonetheless, Article 422 of the Civil Code states that "[p]roperty of public dominion, when no longer intended for public use or
for public service, shall form part of the patrimonial property of the State." It is this provision that controls how public dominion
property may be converted into patrimonial property susceptible to acquisition by prescription. After all, Article 420 (2) makes
clear that those property "which belong to the State, without being for public use, and are intended for some public service or
for the development of the national wealth" are public dominion property. For as long as the property belongs to the State,
although already classified as alienable or disposable, it remains property of the public dominion if when it is "intended for
some public service or for the development of the national wealth."

Accordingly, there must be an express declaration by the State that the public dominion property is no longer intended for
public service or the development of the national wealth or that the property has been converted into patrimonial. Without
such express declaration, the property, even if classified as alienable or disposable, remains property of the public dominion,
pursuant to Article 420(2), and thus incapable of acquisition by prescription. It is only when such alienable and disposable lands
are expressly declared by the State to be no longer intended for public service or for the development of the national wealth
that the period of acquisitive prescription can begin to run. Such declaration shall be in the form of a law duly enacted by
Congress or a Presidential Proclamation in cases where the President is duly authorized by law.23 (Emphasis supplied)

In Republic v. Rizalvo,24 the Court deemed it appropriate to reiterate the ruling in Malabanan, viz:

On this basis, respondent would have been eligible for application for registration because his claim of ownership and
possession over the subject property even exceeds thirty (30) years. However, it is jurisprudentially clear that the thirty (30)-
year period of prescription for purposes of acquiring ownership and registration of public land under Section 14 (2) of P.D. No.
1529 only begins from the moment the State expressly declares that the public dominion property is no longer intended for
public service or the development of the national wealth or that the property has been converted into patrimonial. x x
x.25 (Citation omitted and emphasis ours)

Accordingly, although lands of the public domain that are considered patrimonial may be acquired by prescription under
Section 14(2) of P.D. No. 1529, before acquisitive prescription could commence, the property sought to be registered must not
only be classified as alienable and disposable; it must also be declared by the State that it is no longer intended for public use,
public service or the development of the national wealth. Thus, absent an express declaration by the State, the land remains to
be property of public dominion.26

The Court finds no evidence of any official declaration from the state attesting to the patrimonial character of the subject
property. Cortez failed to prove that acquisitive prescription has begun to run against the State, much less that he has acquired
title to the subject property by virtue thereof. It is of no moment that Cortez and his predecessors-in-interest have been in
possession of the subject property for 57 years at the time he applied for the registration of title thereto. "[l]t is not the
notorious, exclusive and uninterrupted possession and occupation of an alienable and disposable public land for the mandated
periods that converts it to patrimonial. The indispensability of an official declaration that the property is now held by the State
in its private capacity or placed within the commerce of man for prescription to have any effect against the State cannot be
overemphasized. "27

WHEREFORE, in consideration of the foregoing disquisitions, the instant petition is GRANTED. The Decision dated February 17,
2009 of the Court of Appeals in CA-G.R. CV No. 87505, which affirmed the Decision dated February 7, 2006 of the Regional Trial
Court of Pasig City, Branch 68, in LRC Case No. N-11496, is hereby REVERSED and SET ASIDE. The Application for Registration of
Emmanuel C. Cortez in LRC Case No. N-11496 is DENIED for lack of merit.

SO ORDERED.

BIENVENIDO L. REYES
Associate Justice

G.R. No. 191590 April 21, 2014

REPUBLIC OF THE PHILIPPINES, Petitioner,


vs.
TRANSUNION CORPORATION, Respondent.
DECISION

PERLAS-BERNABE, J.:

Assailed in this petition for review on certiorari1 are the Decision2 dated October 9, 2009 and the Resolution3dated March 10,
2010 of the Court of Appeals (CA) in CA-G.R. SP No,. 106544 which set aside the Order4 dated August 14, 2008 of the Regional
Trial Court of Imus, Cavite, Branch 22 (RTC) denying the motion to dismiss filed by respondent Transunion Corporation
(Transunion) in Civil Case No. 2085-08.

The Facts

On April 30, 1999, Leticia Salamat (Salamat) filed an Application to Purchase Friar Lands, 5 specifically Lot No. 5741 of the Imus
Estate (Lot No. 5741), with the Community Environment and Natural Resources Office (CENRO) of the Department of
Environment and Natural Resources (DENR).6 Her application was subsequently indorsed to the Land Management Bureau
(LMB) for final action.7 Thereafter, Salamat was informed that Lot No. 5741 was already covered by Transfer Certificate of Title
(TCT) No. T-6167408 in the name of Transunion.9 This prompted Salamat to file, on June 27, 2000, a Protest10 against
Transunion with the LMB , docketed as LMB Case No. 114, alleging that TCT No. T-616740 was obtained through fraud
considering that no deed of conveyance was issued by the LMB for Lot No. 5741 in the name of any person.11 In this relation,
Salamat averred that she and her family had been in continuous possession and occupation of the said lot since time
immemorial and had even introduced improvements thereon. She likewise stated that it was only after the LMB favorably
endorsed her application, that it was discovered that Lot No. 5741 was already covered by TCT No. T-616740.12

On September 13, 2000, LMB OIC-Director Ernesto D. Adobo, Jr. (Director Adobo) issued Special Order No. 2000-175,
designating Atty. Rogelio C. Mandar (Atty. Mandar) and one Carlito Manga, Jr. to conduct a formal investigation in order to
determine the veracity of the allegations contained in Salamat’s protest pursuant to Lands Office Circular No. 68 (LC 68). 13

On November 8, 2000, Transunion filed with the LMB a motion to dismiss, alleging that Salamat had no legal personality to
attack the validity of Transunion's title, and that it is the RTC which has jurisdiction to try and decide cases involving
cancellation of titles.14 On February 8, 2001, Director Adobo denied the motion to dismiss and directed Atty. Mandar to
proceed with the investigation.15

After due proceedings, Atty. Mandar issued an investigation report16 dated July 8, 2003 (investigation report) addressed to "The
Director Thru the OIC-Chief Legal Division, Lands Management Bureau,"17 recommending that steps be taken before a
competent court of justice for the annulment of TCT No. T-616740 and the reversion of Lot No. 5741 to the government.18 The
recommendation was adopted by the Legal Division in its memorandum19dated November 2003 addressed to the Director,
which was later approved by LMB Director Concordio D. Zuñiga (LMB Director).20

Neither Salamat nor Transunion were furnished copies of the investigation report or memorandum.21

On April 20, 2004, the DENR transmitted to the Office of the Solicitor General (OSG) the entire records of LMB Case No.
114.22 Accordingly, a complaint for cancellation of title and/or reversion, docketed as Civil Case No. 2085-08 (reversion
complaint), was filed by herein petitioner the Republic of the Philippines (Republic) against Transunion and its predecessors-in-
interest, with the RTC.23

In response, Transunion filed a motion to dismiss24 on the ground that the filing of the reversion complaint was premature.
Specifically, it argued that a condition precedent for the filing of the complaint had not been complied with – that is, the failure
of the LMB to notify Transunion of its recommendation in the investigation report – thereby depriving it the opportunity to
seek a reconsideration or an appeal of the same, and ultimately resulting in a failure to exhaust administrative remedies.
Hinged on the foregoing theory, Transunion further claimed that the reversion complaint stated no cause of action.

The RTC Ruling

In an Order25 dated August 14, 2008, the RTC denied Transunion's motion to dismiss.
It held that the investigation report was merely a recommendation for a "possible action that should be taken" by the LMB
Director.26 Accordingly, Atty. Mandar’s actions were not in the exercise of a quasi-judicial function, hence, not subject to a
motion for reconsideration or appeal. It is in this regard that the RTC concluded there was any failure to comply with a
condition precedent.27

Relatedly, the RTC ruled that the Republic’s reversion complaint did state a cause of action based on its examination of the
allegations and arguments stated therein.28

Dissatisfied, Transunion elevated the matter on certiorari.29

The CA Ruling

In a Decision30 dated October 9, 2009, the CA reversed the RTC's ruling, observing that no decision was rendered in LMB Case
No. 114 and that Transunion was denied the right to be informed of the DENR's official action as well as the opportunity to
contest said action. As such, it pronounced that the filing of the Republic’s reversion complaint was premature and that the
latter’s failure to exhaust administrative remedies was fatal to its cause of action.31

At odds with the CA's Decision, the Republic filed a motion for reconsideration,32 which was, however, denied by the CA in a
Resolution33 dated March 10, 2010, hence, this petition.

The Issue Before the Court

The sole issue for the Court's resolution is whether or not the CA correctly granted Transunion’s petition for certiorari against
the RTC's order denying the latter’s motion to dismiss.

The Court's Ruling

The petition is meritorious.

An order denying a motion to dismiss is an interlocutory order which neither terminates nor finally disposes of a case as it
leaves something to be done by the court before the case is finally decided on the merits. Thus, as a general rule, the denial of a
motion to dismiss cannot be questioned in a special civil action for certiorari which is a remedy designed to correct errors of
jurisdiction and not errors of judgment. However, when the denial of the motion to dismiss is tainted with grave abuse of
discretion, the grant of the extraordinary remedy of certiorari may be justified. By grave abuse of discretion is meant such
capricious and whimsical exercise of judgment that is equivalent to lack of jurisdiction. The abuse of discretion must be grave as
where the power is exercised in an arbitrary or despotic manner by reason of passion or personal hostility, and must be so
patent and gross as to amount to an evasion of positive duty or to a virtual refusal to perform the duty enjoined by or to act at
all in contemplation of law.34

In the present case, the Court finds that the RTC did not commit any grave abuse of discretion in denying Transunion’s motion
to dismiss considering that the latter’s further reconsideration or appeal of the investigation report was not a condition
precedent to the filing of the Republic’s reversion complaint. As such, there was no violation of the rule on exhaustion of
administrative remedies nor can it be said that the reversion complaint stated no cause of action.

To elaborate, the rule on exhaustion of administrative remedies provides that if a remedy within the administrative machinery
can still be resorted to by giving the administrative officer concerned every opportunity to decide on a matter that comes
within his jurisdiction, then such remedy should be exhausted first before the court’s judicial power can be sought. The
underlying principle of the rule rests on the presumption that the administrative agency, if afforded a complete chance to pass
upon the matter will decide the same correctly.35

Transunion reiterates that the Republic’s reversion complaint should be dismissed on the ground that it was not notified of the
investigation report, recommending that steps be taken before a competent court of justice for the annulment of TCT No. T-
616740 and the reversion of Lot No. 5741 to the government. It argues that it should have been notified of said report and
recommendation so that it would have been able to contest the same on reconsideration or on appeal. Without having been
able to avail of these remedies, Transunion decries a violation of the rule on exhaustion of administrative remedies and,
perforce, prays that the Republic’s reversion complaint be dismissed.

Transunion is mistaken.

As may be gleaned from the records,36 the LMB proceeding subject of Transunion’s motion to dismiss was merely investigative
in nature since it was conducted as a fact-finding/recommendatory procedure, meant only to determine whether or not the
LMB Director should initiate reversion proceedings. This proceeding was taken under LC 68, captioned as "Investigation of
Claims and Conflicts."37 Section 15 of LC 68, which states the parameters to be observed regarding the report and
recommendation resulting from the said investigation, is bereft of any indication that the remedies of reconsideration or a
further appeal is available to a party disagreeing with the same, viz.:

SEC. 15. Report of Investigation. – Within 30 days from the date of termination of the investigation, the hearing officer
concerned shall render his report on the case to the Regional Executive Director. He shall forward together with his report the
complete records of the proceedings, evidence of the parties and such other papers, documents and record relevant thereto.

The report of the investigation should contain the following:

1. Caption and title of the case;

2. Statement as to how the case arose and by virtue of whose authority investigation was conducted;

3. Statement that notices have been sent to parties and how they were notified;

4. Statement as to when and where formal investigation was conducted;

5. Parties appearing thereat including the counsel representing them, if any, and their addresses;

6. Findings in the ocular inspection including the description of improvements and sketch of the land showing the portion
contested and statement that efforts had been exerted to settle the case amicably between the parties;

7. Summary of the testimony of the parties and witnesses and enumeration and substance of the documentary evidence
submitted by them;

8. Observation on the case including the demeanor of the persons who testified thereat;

9. Recommendations.

The report must be prepared immediately after the hearing while the matter is still fresh in the investigator’s mind. In no case
shall such report be a brief in support of one of the parties or contain a discussion of the law applicable to the case. The
investigator shall present only the facts as he gathered them at the investigation. 38 (Emphases supplied)

Transunion confuses the investigation report and the recommendation made therein with an action of the LMB Regional
Executive Director found in Section 3.1 of the Manual on Settlement of Land Disputes39 (Land Disputes Manual) characterized
as follows:

3.1 Matters covered by decisions or orders of the Regional Executive Director.

All actions of the Regional Executive Director in approving, rejecting; reinstating or cancelling a public land application, or
deciding a conflict, dismissing a claim or determining any matters in relation thereto, shall be published in the form of a judicial
decision or order. All parties concerned or their attorneys or representatives and the Central Office, Lands Management Bureau
shall be furnished copies of the decision or order.40 (Emphases and underscoring supplied)
The distinctions between an investigative function – such as that taken by the LMB in this case – and an adjudicative function –
such as that described in Section 3.1 above – have been extensively discussed by the Court in the case of Cariño v. Commission
on Human Rights,41 to wit:

"Investigate," commonly understood, means to examine, explore, inquire or delve or probe into, research on, study. The
dictionary definition of "investigate" is "to observe or study closely; inquire into systematically: ‘to search or inquire into’ x x x
to subject to an official probe x x x: to conduct an official inquiry." The purpose of [an] investigation, of course is to discover, to
find out, to learn, obtain information. Nowhere included or intimated is the notion of settling, deciding or resolving a
controversy involved in the facts inquired into by application of the law to the facts established by the inquiry.

The legal meaning of "investigate" is essentially the same: "(t)o follow up step by step by patient inquiry or observation. To
trace or track; to search into; to examine and inquire into with care and accuracy; to find out by careful inquisition;
examination; the taking of evidence; a legal inquiry;" "to inquire; to make an investigation," "investigation" being in turn
described as "(a)n administrative function, the exercise of which ordinarily does not require a hearing. 2 Am J2d Adm L Sec.
257; x x x an inquiry, judicial or otherwise, for the discovery and collection of facts concerning a certain matter or matters."

"Adjudicate," commonly or popularly understood, means to adjudge, arbitrate, judge, decide, determine, resolve, rule on,
settle. The dictionary defines the term as "to settle finally (the rights and duties of parties to a court case) on the merits of
issues raised: x x x to pass judgment on:

settle judicially: x x x act as judge." And "adjudge" means "to decide or rule upon as a judge or with judicial or quasi-judicial
powers: x x x to award or grant judicially in a case of controversy x x x."

In the legal sense, "adjudicate" means: "To settle in the exercise of judicial authority. To determine finally. Synonymous with
adjudge in its strictest sense;" and "adjudge" means: "To pass on judicially, to decide, settle, or decree, or to sentence or
condemn. x x x Implies a judicial determination of a fact, and the entry of a judgment."42(Emphases supplied)

Based on Section 3.1 of the Land Disputes Manual as above-cited, it is clear that it is the action of the Regional Executive
Director in "approving, rejecting; reinstating or cancelling a public land application, or deciding a conflict, dismissing a claim or
determining any matters in relation thereto" which is required to be "published in the form of a judicial decision or order," and
from which the remedies of reconsideration and appeal may be taken pursuant to Section 3.2 of the Land Disputes Manual, viz.:

3.2 Appeal from decision or order of the Regional Executive Director

An appeal from a decision or order of the Regional Executive Director to the Secretary of Environment and Natural Resources
shall be within a period of thirty (30) days to be counted from the date the interested party received the notice thereof unless a
motion for reconsideration is filed within the said period, in which case, appeal shall be made within thirty (30) days from his
receipt of notice of the order or decision of the Regional Executive Director disposing of the motion [for] reconsideration. The
notice of appeal may be delivered or sent to (1) the Community Environment and Natural Resources Officer or provincial
Environment and Natural Resources Officer, (2) the Regional Director deciding the case, (3) the Secretary or Undersecretary of
Environment and Natural Resources. On receipt thereof, the Community Environment and Natural Resources Officer or
Provincial Environment and Natural Resources Officer concerned shall note thereon the date when it is received by him and
shall forward the same without delay to the Secretary of Environment and Natural Resources. In case the appeal is delivered or
sent to the Director of Lands, he shall forward the same together with the official records pertinent to the case to the Secretary
of Environment and Natural Resources. In case the appeal is sent directly to the Secretary of Environment and Natural
Resources, the Regional Executive Director shall be served with a notice of appeal. The time of filing the appeal with any one of
the said officers, as specified in Section 16 thereof, shall be considered as the time when the appeal is taken. 43 (Emphases and
underscoring supplied)

Said course of action is that which may be considered as a form of "adjudication," resulting as it would in the settlement of a
public land application, or a decision on a public land conflict or claim. Given its nature, the Land Disputes Manual then requires
that it be published in the form of a judicial decision or order and, concomitantly, be subject to further reconsideration and/or
appeal. This action is clearly different from the LMB proceeding subject of this case which, as earlier stated, is merely
investigative in nature. As further explained by the Republic in its petition, "[t]he investigation carried out by the Director of
Lands merely determines the propriety of initiating reversion proceedings and is an internal procedure within the exclusive
discretion of the LMB."44 With this in mind, the latter proceeding and the recommendation reached thereby cannot then be
considered to be governed by Sections 3.1 and 3.2 of the Land Disputes Manual which respectively provide the requirement of
notice and the remedies of reconsideration or appeal. Corollarily, since these administrative remedies were not available to
Transunion against the investigation report and recommendation, there was thus no violation of the rule on exhaustion of
administrative remedies. As such, Transunion’s claim that a condition precedent was left unfulfilled was properly debunked by
the RTC.

Further, barring any violation of the doctrine of exhaustion of administrative remedies as above-discussed, Transunion’s
assertion on the reversion complaint’s purported failure to state a cause of action (or properly speaking, the Republic’s lack of
cause of action)45 – hinged as it is solely on the same theory - was also properly denied.1âwphi1

Finally, the Court finds that there was no violation of Transunion's right to administrative due process since, as the Republic
pointed out, not only did it file an answer, but it also presented its evidence and formally offered the .same.46 It is well-
established that the touchstone of due process is the opportunity to be heard. 47 This Transunion was unquestionably afforded
in this case, despite having been denied the remedies of reconsideration and appeal which, however, remain unavailable, either
by statute or regulation, against the investigation report and recomme'ndation assailed herein. At any rate, lack of
administrative due process, on the assumption of its truth, is not a ground for a motion to dismiss;48 hence, the RTC's ruling was
altogether proper.

For the reasons above-stated, the Court therefore concludes that the RTC did not gravely abuse its discretion in denying
Transunion 's motion to dismiss against the Republic's reversion complaint. As such, the CA committed a reversible error in
granting Transunion's petition for certiorari, warranting the reversal of its Decision.

WHEREFORE the petition is GRANTED. The Decision dated October 9, 2009 and the Resolution dated March 10, 2010 of the
Court of Appeals in .CA-G.R. SP No. 106544 are hereby REVERSED and SET ASIDE.

SO ORDERED.

ESTELA M. PERLAS-BERNABE
Associate Justice

[G.R. No. 135385. December 6, 2000]

ISAGANI CRUZ and CESAR EUROPA, petitioners, vs. SECRETARY OF ENVIRONMENT AND NATURAL RESOURCES, SECRETARY OF
BUDGET AND MANAGEMENT and CHAIRMAN and COMMISSIONERS OF THE NATIONAL COMMISSION ON INDIGENOUS
PEOPLES, respondents.

PER CURIAM:

Petitioners Isagani Cruz and Cesar Europa brought this suit for prohibition and mandamus as citizens and taxpayers, assailing
the constitutionality of certain provisions of Republic Act No. 8371 (R.A. 8371), otherwise known as the Indigenous Peoples
Rights Act of 1997 (IPRA), and its Implementing Rules and Regulations (Implementing Rules).

In its resolution of September 29, 1998, the Court required respondents to comment.[1] In compliance, respondents
Chairperson and Commissioners of the National Commission on Indigenous Peoples (NCIP), the government agency created
under the IPRA to implement its provisions, filed on October 13, 1998 their Comment to the Petition, in which they defend the
constitutionality of the IPRA and pray that the petition be dismissed for lack of merit.

On October 19, 1998, respondents Secretary of the Department of Environment and Natural Resources (DENR) and Secretary of
the Department of Budget and Management (DBM) filed through the Solicitor General a consolidated Comment. The Solicitor
General is of the view that the IPRA is partly unconstitutional on the ground that it grants ownership over natural resources to
indigenous peoples and prays that the petition be granted in part.

On November 10, 1998, a group of intervenors, composed of Sen. Juan Flavier, one of the authors of the IPRA, Mr. Ponciano
Bennagen, a member of the 1986 Constitutional Commission, and the leaders and members of 112 groups of indigenous
peoples (Flavier, et. al), filed their Motion for Leave to Intervene. They join the NCIP in defending the constitutionality of IPRA
and praying for the dismissal of the petition.

On March 22, 1999, the Commission on Human Rights (CHR) likewise filed a Motion to Intervene and/or to Appear as Amicus
Curiae. The CHR asserts that IPRA is an expression of the principle of parens patriae and that the State has the responsibility to
protect and guarantee the rights of those who are at a serious disadvantage like indigenous peoples. For this reason it prays
that the petition be dismissed.

On March 23, 1999, another group, composed of the Ikalahan Indigenous People and the Haribon Foundation for the
Conservation of Natural Resources, Inc. (Haribon, et al.), filed a motion to Intervene with attached Comment-in-
Intervention. They agree with the NCIP and Flavier, et al. that IPRA is consistent with the Constitution and pray that the petition
for prohibition and mandamus be dismissed.

The motions for intervention of the aforesaid groups and organizations were granted.

Oral arguments were heard on April 13, 1999. Thereafter, the parties and intervenors filed their respective memoranda in
which they reiterate the arguments adduced in their earlier pleadings and during the hearing.

Petitioners assail the constitutionality of the following provisions of the IPRA and its Implementing Rules on the ground that
they amount to an unlawful deprivation of the States ownership over lands of the public domain as well as minerals and other
natural resources therein, in violation of the regalian doctrine embodied in Section 2, Article XII of the Constitution:

(1) Section 3(a) which defines the extent and coverage of ancestral domains, and Section 3(b) which, in turn, defines ancestral
lands;

(2) Section 5, in relation to section 3(a), which provides that ancestral domains including inalienable public lands, bodies of
water, mineral and other resources found within ancestral domains are private but community property of the indigenous
peoples;

(3) Section 6 in relation to section 3(a) and 3(b) which defines the composition of ancestral domains and ancestral lands;

(4) Section 7 which recognizes and enumerates the rights of the indigenous peoples over the ancestral domains;

(5) Section 8 which recognizes and enumerates the rights of the indigenous peoples over the ancestral lands;

(6) Section 57 which provides for priority rights of the indigenous peoples in the harvesting, extraction, development or
exploration of minerals and other natural resources within the areas claimed to be their ancestral domains, and the right to
enter into agreements with nonindigenous peoples for the development and utilization of natural resources therein for a
period not exceeding 25 years, renewable for not more than 25 years; and

(7) Section 58 which gives the indigenous peoples the responsibility to maintain, develop, protect and conserve the ancestral
domains and portions thereof which are found to be necessary for critical watersheds, mangroves, wildlife sanctuaries,
wilderness, protected areas, forest cover or reforestation.[2]

Petitioners also content that, by providing for an all-encompassing definition of ancestral domains and ancestral lands which
might even include private lands found within said areas, Sections 3(a) and 3(b) violate the rights of private landowners. [3]
In addition, petitioners question the provisions of the IPRA defining the powers and jurisdiction of the NCIP and making
customary law applicable to the settlement of disputes involving ancestral domains and ancestral lands on the ground that
these provisions violate the due process clause of the Constitution.[4]

These provisions are:

(1) sections 51 to 53 and 59 which detail the process of delineation and recognition of ancestral domains and which vest on the
NCIP the sole authority to delineate ancestral domains and ancestral lands;

(2) Section 52[i] which provides that upon certification by the NCIP that a particular area is an ancestral domain and upon
notification to the following officials, namely, the Secretary of Environment and Natural Resources, Secretary of Interior and
Local Governments, Secretary of Justice and Commissioner of the National Development Corporation, the jurisdiction of said
officials over said area terminates;

(3) Section 63 which provides the customary law, traditions and practices of indigenous peoples shall be applied first with
respect to property rights, claims of ownership, hereditary succession and settlement of land disputes, and that any doubt or
ambiguity in the interpretation thereof shall be resolved in favor of the indigenous peoples;

(4) Section 65 which states that customary laws and practices shall be used to resolve disputes involving indigenous peoples;
and

(5) Section 66 which vests on the NCIP the jurisdiction over all claims and disputes involving rights of the indigenous peoples.[5]

Finally, petitioners assail the validity of Rule VII, Part II, Section 1 of the NCIP Administrative Order No. 1, series of 1998, which
provides that the administrative relationship of the NCIP to the Office of the President is characterized as a lateral but
autonomous relationship for purposes of policy and program coordination. They contend that said Rule infringes upon the
Presidents power of control over executive departments under Section 17, Article VII of the Constitution.[6]

Petitioners pray for the following:

(1) A declaration that Sections 3, 5, 6, 7, 8, 52[I], 57, 58, 59, 63, 65 and 66 and other related provisions of R.A. 8371 are
unconstitutional and invalid;

(2) The issuance of a writ of prohibition directing the Chairperson and Commissioners of the NCIP to cease and desist from
implementing the assailed provisions of R.A. 8371 and its Implementing Rules;

(3) The issuance of a writ of prohibition directing the Secretary of the Department of Environment and Natural Resources to
cease and desist from implementing Department of Environment and Natural Resources Circular No. 2, series of 1998;

(4) The issuance of a writ of prohibition directing the Secretary of Budget and Management to cease and desist from disbursing
public funds for the implementation of the assailed provisions of R.A. 8371; and

(5) The issuance of a writ of mandamus commanding the Secretary of Environment and Natural Resources to comply with his
duty of carrying out the States constitutional mandate to control and supervise the exploration, development, utilization and
conservation of Philippine natural resources.[7]

After due deliberation on the petition, the members of the Court voted as follows:

Seven (7) voted to dismiss the petition. Justice Kapunan filed an opinion, which the Chief Justice and Justices Bellosillo,
Quisumbing, and Santiago join, sustaining the validity of the challenged provisions of R.A. 8371. Justice Puno also filed a
separate opinion sustaining all challenged provisions of the law with the exception of Section 1, Part II, Rule III of NCIP
Administrative Order No. 1, series of 1998, the Rules and Regulations Implementing the IPRA, and Section 57 of the IPRA which
he contends should be interpreted as dealing with the large-scale exploitation of natural resources and should be read in
conjunction with Section 2, Article XII of the 1987 Constitution. On the other hand, Justice Mendoza voted to dismiss the
petition solely on the ground that it does not raise a justiciable controversy and petitioners do not have standing to question
the constitutionality of R.A. 8371.

Seven (7) other members of the Court voted to grant the petition. Justice Panganiban filed a separate opinion expressing the
view that Sections 3 (a)(b), 5, 6, 7 (a)(b), 8, and related provisions of R.A. 8371 are unconstitutional. He reserves judgment on
the constitutionality of Sections 58, 59, 65, and 66 of the law, which he believes must await the filing of specific cases by those
whose rights may have been violated by the IPRA. Justice Vitug also filed a separate opinion expressing the view that Sections
3(a), 7, and 57 of R.A. 8371 are unconstitutional.Justices Melo, Pardo, Buena, Gonzaga-Reyes, and De Leon join in the separate
opinions of Justices Panganiban and Vitug.

As the votes were equally divided (7 to 7) and the necessary majority was not obtained, the case was redeliberated
upon. However, after redeliberation, the voting remained the same.Accordingly, pursuant to Rule 56, Section 7 of the Rules of
Civil Procedure, the petition is DISMISSED.

Attached hereto and made integral parts thereof are the separate opinions of Justices Puno, Vitug, Kapunan, Mendoza, and
Panganiban.

SO ORDERED.

G.R. No. L-12691 February 27, 1959

SIMEON T. DAGDAG, plaintiff-appellee,


vs.
VICENTE NEPOMUCENO, ET AL., defendants-appellants.

Jesus Paredes for appellee.


Mariano Sta. Romana for appellants.

BENGZON, J.:

Forwarded by the Court of Appeals, this lawsuit coming from Nueva Ecija, concern a small parcel of land. Submitted for decision
below upon a stipulation of facts, it raises legal questions only.

A portion of Lot No. 3786, Cabanatuan Cadaster (admittedly alienable or disposable public land way back in 1916) is covered by
Sales Patent No. 251 issued to Margarita Juanson, and also by lease No. 49 executed by the Bureau of Lands in favor of Andres
de Vera. The overlapping was recently discovered, and their successors in interest now litigate for possession and/or
ownership.

The Sales Patent was inscribed in the office of the Register of Deeds on July 11, 1927, and Original Certificate of Title No. 68 was
accordingly issued in the same of Margarita Juanson, who later sold the land of Remegio Juanson Bautista (1928), who in turn
sold it to Balarian Incorporated (1929). In May 1950, Simeon T. Dagdag bought it from Balarin, Inc. After every sale, the
corresponding Transfer Certificate of Title was given out.

On the other hand, the lease to De Vera signed in June 1916 covered adjoining land of a bigger area. It was transferred by him
to Regino Nepomuceno. Originally for a 25-years period expiring on June 30, 1941, it was extended for another like period in
1949. Dagdag's title, and those of his predecessors contained no annotation of such lease, of which neither he nor they any
knowledge.

After purchasing the land, Simeon T. Dagdag had it relocated and the portion in question turned out to be in possession of the
heirs of Regino Nepomuceno, appellants herein allegedly by virtue of the lease. The latter refused to surrender it, even in the
face of Dagdag's patent and title, and despite the Director of Lands' administrative determination in February 1953, practically
holding that their contract of lease did not, could not and should not extend to the area granted to Dagdag's predecessors.
Hence, this judicial proceeding instituted by Dagdag in the Nueva Ecija court of first instance, wherein he was declared to be the
owner of the whole Lot 3786 and entitled to the products thereof. The Honorable Jose N. Leutrio, Judge, explained that "the
sales patent issued in the name of Margarita Juanson having been registered with the office of the Register of Deeds, and title
having been issued by the Register of Deeds in the name of Margarita Juanson, Lot 3786 was thereafter brought under the
operation of the Land Registration Act. The title issued in the name of Margarita Juanson, Original Certificate of Title No. 68 was
free from all liens and incumbrances. This land was transferred successively, until it was acquired by the plaintiff herein, and the
certificate of title was issued in his name free from any lien or encumbrances, and free from the claim of Regino Nepomuceno
as losses. The plaintiffs herein cannot, therefore, be bound by the fact that Lot 3786 is within the lease of Andres de Vera which
had been transferred to Regino Nepomuceno, the father and predecessor of the defendants herein. The said lease not having
been annotated on the certificate of title, and it not having been neither proved or alleged that the plaintiff had purchased the
land knowing that Lot 3786 is a portion of the land leased to Andres de Vera which had been acquired by the defendant's
predecessors-in-interest, it cannot prejudice the plaintiff who is presumed to be an innocent purchaser for value. The fact that
the lease in favor of Andres de Vera had been registered, cannot bind and prejudice the plaintiff for Lot 3786 being a registered
land, he need not go farther than the title."

The above observations deserve our approval. They conform with our decisions on indefeasibility of public land patents when
registered in the corresponding Register of Deeds Office.1 We regard these to be veritable Torrens Title subject to no
encumbrances except those stated therein, plus those specified by the status (lease is not one of them).

In addition to the above reason given by his Honor, it should be remembered that when the lease was renewed in 1949, the
portion in question was no longer public land subject to the disposition of the Director of Lands because it had already been
granted to Margarita Juanson and had become private property; therefore, it could not have been included in the renewal of
such lease of public land.

Defendant's position may be summed up, in their own words, as follows:

When the contract of lease of the predecessor of the defendants was duly issued and registered in the office of the register of
deeds of Nueva Ecija, and when the patent for the certificate of sale in favor of the predecessor of the plaintiff was issued and
registered in the said register of deeds of Nueva Ecija, both documents have the force and effect of registered properties under
the land Registration Act as provided for in . . . (section 122 of the Land Registration Law). . . .

As the titles of the parties have come under the operation of the Land Registration Act, and in case of overlapping titles, the
older title should prevail. The title of the defendants was issued and registered on June 14, 1916. The title of the plaintiff was
registered on August 5, 1927. The title of the defendants should, therefore prevail, and they should have been declared the
owners of the land in question. (pp. 8-9 Appellants Brief) [Emphasis Ours].

The flaw in their argument lies in the assumption that their lease contract constituted a "title", or deed or conveyance within
the meaning of section 122, which for convenience is quoted below:

Whenever public lands in the Philippine Islands belonging to the Government of the United States or the Government of the
Philippine Islands are alienated, granted, or conveyed to persons or to public or private corporations, the same shall be brought
forthwith under the operation of this Act and shall become registered lands. It shall be the duty of the officials issuing the
instrument of alienation, grant, or conveyance in behalf of the Government to cause such instrument, before its delivery to the
grantee, to be filed with the register of deeds for the province where the land lies and to be there registered like other deeds
and conveyance, whereupon a certificate shall be entered as in other cases of registered land, and an owner's duplicate issue to
the grantee. The deed, grant, or instrument of conveyance from the Government to the grantee shall not take effect as a
conveyance or bind the land, . . . After due registration and issue of the certificate and owner's duplicate such shall be
registered land for all purposes under this Act.

Upon carefully reading the above, we think it clear that the documents mentioned, wherein lands are "alienated, granted, or
conveyed", are documents transferring ownership — not documents of lease, transferring mere possession. Observe especially
that the statue directs the issuance to the grantee of "an owner's duplicate certificate". Appellants may not, therefore, assert
a title just as good-so they claim-as appellee's and older besides. So, the Torrens Title of appellee must prevail.
Judgment affirmed, with costs against appellants.

Paras, C.J., Padilla, Montemayor, Reyes, A., Bautista Angelo, Labrador, Concepcion and Endencia, JJ., concur.

CHAVEZ V. PUBLIC ESTATE AUTHORITY

FACTS:

From the time of Marcos until Estrada, portions of Manila Bay were being reclaimed. A law was passed creating the Public
Estate Authority which was granted with the power to transfer reclaimed lands. Now in this case, PEA entered into a Joint
Venture Agreement with AMARI, a private corporation. Under the Joint Venture Agreement between AMARI and PEA, several
hectares of reclaimed lands comprising the Freedom Islands and several portions of submerged areas of Manila Bay were going
to be transferred to AMARI .

ISSUE:

Whether or not the stipulations in the Amended JVA for the transfer to AMARI of lands, reclaimed or to be reclaimed, violate
the Constitution

RULING: YES!

Under the Public Land Act (CA 141, as amended), reclaimed lands are classified as alienable and disposable lands of the public
domain Section 3 of the Constitution: Alienable lands of the public domain shall be limited to agricultural lands. Private
corporations or associations may not hold such alienable lands of the public domain except by lease The 157.84 hectares of
reclaimed lands comprising the Freedom Islands, now covered by certificates of title in the name of PEA, are alienable lands of
the public domain. PEA may lease these lands to private corporations but may not sell or transfer ownership of these lands to
private corporations. PEA may only sell these lands to Philippine
citizens, subject to the ownership limitations in the 1987 Constitution and existing laws. Clearly, the Amended JVA violates
glaringly Sections 2 and 3, Article XII of the 1987 Constitution. Under Article 1409 of the Civil Code, contracts whose “object or
purpose is contrary to law,” or whose “object is outside the commerce of men,” are “inexistent and void from the beginning.”
The Court must perform its duty to defend and uphold the Constitution, and therefore declares the Amended JVA null and void
ab initio.

CHAVEZ V. PUBLIC ESTATES AUTHORITY

384 SCRA 152

FACTS:

President Marcos through a presidential decree created PEA, which was


tasked with the development, improvement, and acquisition, lease, and sale of all kinds of lands. The then president also
transferred to PEA the foreshore and offshore lands of Manila Bay under the Manila-Cavite Coastal
Road and Reclamation Project.
Thereafter, PEA was granted patent to the reclaimed areas of land and then, years later, PEA entered into a JVA with
AMARI for the development of the Freedom Islands. These two entered into a joint venture in the absence of any public
bidding.

Later, a privilege speech was given by Senator President Maceda denouncing the JVA as the grandmother of all
scams. An investigation was conducted and it was concluded that the lands that PEA was conveying to
AMARI were lands of the public domain; the certificates of title over the
Freedom Islands were void; and the JVA itself was illegal. This prompted Ramos to form an investigatory committee on the
legality of the JVA.

Petitioner now comes and contends that the government stands to lose
billions by the conveyance or sale of the reclaimed areas to AMARI. He also asked for the full disclosure of the
renegotiations happening between the parties.

ISSUE:

W/N stipulations in the amended JVA for the transfer to AMARI of the lands, reclaimed or to be reclaimed, violate the
Constitution.

HELD:

The ownership of lands reclaimed from foreshore and submerged areas is rooted in the Regalian doctrine, which holds that the
State owns all lands and waters of the public domain.

The 1987 Constitution recognizes the Regalian doctrine. It declares that all
natural resources are owned by the State and except for alienable
agricultural lands of the public domain, natural resources cannot be alienated.

The Amended JVA covers a reclamation area of 750 hectares. Only 157.84 hectares of the 750 hectare reclamation project
have been reclaimed, and the rest of the area are still submerged areas forming part of Manila
Bay. Further, it is provided that AMARI will reimburse the actual costs in reclaiming the areas of land and it will shoulder
the other reclamation costs to be incurred.

The foreshore and submerged areas of Manila Bay are part of the lands of
the public domain, waters and other natural resources and consequently owned by the State. As such, foreshore and
submerged areas shall not be alienable unless they are classified as agricultural lands of the public domain. The mere
reclamation of these areas by the PEA doesn’t convert these inalienable natural resources of the State into alienable and
disposable lands of the public domain. There must be a law or presidential
proclamation officially classifying these reclaimed lands as alienable and
disposable if the law has reserved them for some public or quasi-public use.

G.R. No. 134209 January 24, 2006

REPUBLIC OF THE PHILIPPINES, Petitioner,


vs.
CELESTINA NAGUIAT, Respondent.
DECISION

GARCIA, J.:

Before the Court is this petition for review under Rule 45 of the Rules of Court seeking the reversal of the Decision1 dated May
29, 1998 of the Court of Appeals (CA) in CA-G.R. CV No. 37001 which affirmed an earlier decision2 of the Regional Trial Court at
Iba, Zambales, Branch 69 in Land Registration Case No. N-25-1.

The decision under review recites the factual backdrop, as follows:

This is an application for registration of title to four (4) parcels of land located in Panan, Botolan, Zambales, more particularly
described in the amended application filed by Celestina Naguiat on 29 December 1989 with the Regional Trial Court of
Zambales, Branch 69. Applicant [herein respondent] alleges, inter alia, that she is the owner of the said parcels of land having
acquired them by purchase from the LID Corporation which likewise acquired the same from Demetria Calderon, Josefina
Moraga and Fausto Monje and their predecessors-in-interest who have been in possession thereof for more than thirty (30)
years; and that to the best of her knowledge, said lots suffer no mortgage or encumbrance of whatever kind nor is there any
person having any interest, legal or equitable, or in possession thereof.

On 29 June 1990, the Republic of the Philippines [herein petitioner]. . . filed an opposition to the application on the ground that
neither the applicant nor her predecessors-in interest have been in open, continuous, exclusive and notorious possession and
occupation of the lands in question since 12 June 1945 or prior thereto; that the muniments of title and tax payment receipts of
applicant do not constitute competent and sufficient evidence of a bona-fide acquisition of the lands applied for or of his open,
continuous, exclusive and notorious possession and occupation thereof in the concept of (an) owner; that the applicant’s claim
of ownership in fee simple on the basis of Spanish title or grant can no longer be availed of . . .; and that the parcels of land
applied for are part of the public domain belonging to the Republic of the Philippines not subject to private appropriation.

On 15 October 1990, the lower court issued an order of general default as against the whole world, with the exception of the
Office of the Solicitor General, and proceeded with the hearing of this registration case.

After she had presented and formally offered her evidence . . . applicant rested her case. The Solicitor General, thru the
Provincial Prosecutor, interposed no objection to the admission of the exhibits. Later . . . the Provincial Prosecutor manifest (sic)
that the Government had no evidence to adduce. 3

In a decision4 dated September 30, 1991, the trial court rendered judgment for herein respondent Celestina Naguiat,
adjudicating unto her the parcels of land in question and decreeing the registration thereof in her name, thus:

WHEREFORE, premises considered, this Court hereby adjudicates the parcels of land situated in Panan, Botolan, Zambales,
appearing on Plan AP-03-003447 containing an area of 3,131 square meters, appearing on Plan AP-03-003446 containing an
area of 15,322 containing an area of 15,387 square meters to herein applicant Celestina T. Naguiat, of legal age, Filipino citizen,
married to Rommel Naguiat and a resident of Angeles City, Pampanga together with all the improvements existing thereon and
orders and decrees registration in her name in accordance with Act No. 496, Commonwealth Act No. 14, [should be 141] as
amended, and Presidential Decree No. 1529. This adjudication, however, is subject to the various easements/reservations
provided for under pertinent laws, presidential decrees and/or presidential letters of instructions which should be annotated/
projected on the title to be issued. And once this decision becomes final, let the corresponding decree of registration be
immediately issued. (Words in bracket added)

With its motion for reconsideration having been denied by the trial court, petitioner Republic went on appeal to the CA in CA-
G.R. CV No. 37001.

As stated at the outset hereof, the CA, in the herein assailed decision of May 29, 1998, affirmed that of the trial court, to wit:

WHEREFORE, premises considered, the decision appealed from is hereby AFFIRMED.

SO ORDERED.
Hence, the Republic’s present recourse on its basic submission that the CA’s decision "is not in accordance with law,
jurisprudence and the evidence, since respondent has not established with the required evidence her title in fee simple or
imperfect title in respect of the subject lots which would warrant their registration under … (P.D. 1529 or Public Land Act (C.A.)
141." In particular, petitioner Republic faults the appellate court on its finding respecting the length of respondent’s occupation
of the property subject of her application for registration and for not considering the fact that she has not established that the
lands in question have been declassified from forest or timber zone to alienable and disposable property.

Public forest lands or forest reserves, unless declassified and released by positive act of the Government so that they may form
part of the disposable agricultural lands of the public domain, are not capable of private appropriation.5 As to these assets, the
rules on confirmation of imperfect title do not apply.6 Given this postulate, the principal issue to be addressed turns on the
question of whether or not the areas in question have ceased to have the status of forest or other inalienable lands of the
public domain.

Forests, in the context of both the Public Land Act7 and the Constitution8 classifying lands of the public domain into
"agricultural, forest or timber, mineral lands and national parks," do not necessarily refer to a large tract of wooded land or an
expanse covered by dense growth of trees and underbrush. As we stated in Heirs of Amunategui 9-

A forested area classified as forest land of the public domain does not lose such classification simply because loggers or settlers
have stripped it of its forest cover. Parcels of land classified as forest land may actually be covered with grass or planted to
crops by kaingin cultivators or other farmers. "Forest lands" do not have to be on mountains or in out of the way places. xxx.
The classification is merely descriptive of its legal nature or status and does not have to be descriptive of what the land actually
looks like. xxx

Under Section 2, Article XII of the Constitution,10 which embodies the Regalian doctrine, all lands of the public domain belong to
the State – the source of any asserted right to ownership of land.11 All lands not appearing to be clearly of private dominion
presumptively belong to the State.12 Accordingly, public lands not shown to have been reclassified or released as alienable
agricultural land or alienated to a private person by the State remain part of the inalienable public domain. 13 Under Section 6 of
the Public Land Act, the prerogative of classifying or reclassifying lands of the public domain, i.e., from forest or mineral to
agricultural and vice versa, belongs to the Executive Branch of the government and not the court.14 Needless to stress, the onus
to overturn, by incontrovertible evidence, the presumption that the land subject of an application for registration is alienable or
disposable rests with the applicant.15

In the present case, the CA assumed that the lands in question are already alienable and disposable. Wrote the appellate court:

The theory of [petitioner] that the properties in question are lands of the public domain cannot be sustained as it is directly
against the above doctrine. Said doctrine is a reaffirmation of the principle established in the earlier cases . . . that open,
exclusive and undisputed possession of alienable public land for period prescribed by law creates the legal fiction whereby the
land, upon completion of the requisite period, ipso jure and without the need of judicial or other sanction, ceases to be public
land and becomes private property …. (Word in bracket and underscoring added.)

The principal reason for the appellate court’s disposition, finding a registerable title for respondent, is her and her predecessor-
in-interest’s open, continuous and exclusive occupation of the subject property for more than 30 years. Prescinding from its
above assumption and finding, the appellate court went on to conclude, citing Director of Lands vs. Intermediate Appellate
Court (IAC)16 and Herico vs. DAR,17 among other cases, that, upon the completion of the requisite period of possession, the
lands in question cease to be public land and become private property.

Director of Lands, Herico and the other cases cited by the CA are not, however, winning cards for the respondent, for the simple
reason that, in said cases, the disposable and alienable nature of the land sought to be registered was established, or, at least,
not put in issue. And there lies the difference.

Here, respondent never presented the required certification from the proper government agency or official proclamation
reclassifying the land applied for as alienable and disposable. Matters of land classification or reclassification cannot be
assumed. It calls for proof.18 Aside from tax receipts, respondent submitted in evidence the survey map and technical
descriptions of the lands, which, needless to state, provided no information respecting the classification of the property. As the
Court has held, however, these documents are not sufficient to overcome the presumption that the land sought to be
registered forms part of the public domain.19

It cannot be overemphasized that unwarranted appropriation of public lands has been a notorious practice resorted to in land
registration cases.20 For this reason, the Court has made it a point to stress, when appropriate, that declassification of forest
and mineral lands, as the case may be, and their conversion into alienable and disposable lands need an express and positive
act from the government.21

The foregoing considered, the issue of whether or not respondent and her predecessor-in-interest have been in open, exclusive
and continuous possession of the parcels of land in question is now of little moment. For, unclassified land, as here, cannot be
acquired by adverse occupation or possession; occupation thereof in the concept of owner, however long, cannot ripen into
private ownership and be registered as title.22

WHEREFORE, the instant petition is GRANTED and the assailed decision dated May 29, 1998 of the Court of Appeals in CA-G.R.
CV No. 37001 is REVERSED and SET ASIDE. Accordingly, respondent’s application for original registration of title in Land
Registration Case No. N-25-1 of the Regional Trial Court at Iba, Zambales, Branch 69, is DENIED.

No costs.

G.R. No. L-43938 April 15, 1988

REPUBLIC OF THE PHILIPPINES (DIRECTOR OF FOREST DEVELOPMENT), petitioner,


vs.
HON. COURT OF APPEALS (THIRD DIVISION) and JOSE Y. DE LA ROSA, respondents.

CRUZ, J.:

The Regalian doctrine reserves to the State all natural wealth that may be found in the bowels of the earth even if the land
where the discovery is made be private. 1 In the cases at bar, which have been consolidated because they pose a common issue,
this doctrine was not correctly applied.

These cases arose from the application for registration of a parcel of land filed on February 11, 1965, by Jose de la Rosa on his
own behalf and on behalf of his three children, Victoria, Benjamin and Eduardo. The land, situated in Tuding, Itogon, Benguet
Province, was divided into 9 lots and covered by plan Psu-225009. According to the application, Lots 1-5 were sold to Jose de la
Rosa and Lots 6-9 to his children by Mamaya Balbalio and Jaime Alberto, respectively, in 1964. 2

The application was separately opposed by Benguet Consolidated, Inc. as to Lots 1-5, Atok Big Wedge Corporation, as to
Portions of Lots 1-5 and all of Lots 6-9, and by the Republic of the Philippines, through the Bureau of Forestry Development, as
to lots 1-9. 3

In support of the application, both Balbalio and Alberto testified that they had acquired the subject land by virtue of
prescription Balbalio claimed to have received Lots 1-5 from her father shortly after the Liberation. She testified she was born in
the land, which was possessed by her parents under claim of ownership. 4 Alberto said he received Lots 6-9 in 1961 from his
mother, Bella Alberto, who declared that the land was planted by Jaime and his predecessors-in-interest to bananas, avocado,
nangka and camote, and was enclosed with a barbed-wire fence. She was corroborated by Felix Marcos, 67 years old at the
time, who recalled the earlier possession of the land by Alberto's father. 5 Balbalio presented her tax declaration in 1956 and
the realty tax receipts from that year to 1964, 6 Alberto his tax declaration in 1961 and the realty tax receipts from that year to
1964. 7

Benguet opposed on the ground that the June Bug mineral claim covering Lots 1-5 was sold to it on September 22, 1934, by the
successors-in-interest of James Kelly, who located the claim in September 1909 and recorded it on October 14, 1909. From the
date of its purchase, Benguet had been in actual, continuous and exclusive possession of the land in concept of owner, as
evidenced by its construction of adits, its affidavits of annual assessment, its geological mappings, geological samplings and
trench side cuts, and its payment of taxes on the land. 8

For its part, Atok alleged that a portion of Lots 1-5 and all of Lots 6-9 were covered by the Emma and Fredia mineral claims
located by Harrison and Reynolds on December 25, 1930, and recorded on January 2, 1931, in the office of the mining recorder
of Baguio. These claims were purchased from these locators on November 2, 1931, by Atok, which has since then been in open,
continuous and exclusive possession of the said lots as evidenced by its annual assessment work on the claims, such as the
boring of tunnels, and its payment of annual taxes thereon. 9

The location of the mineral claims was made in accordance with Section 21 of the Philippine Bill of 1902 which provided that:

SEC. 21. All valuable mineral deposits in public lands in the philippine Islands both surveyed and unsurveyed are hereby
declared to be free and open to exploration, occupation and purchase and the land in which they are found to occupation and
purchase by the citizens of the United States, or of said islands.

The Bureau of Forestry Development also interposed its objection, arguing that the land sought to be registered was covered by
the Central Cordillera Forest Reserve under Proclamation No. 217 dated February 16, 1929. Moreover, by reason of its nature,
it was not subject to alienation under the Constitutions of 1935 and 1973. 10

The trial court * denied the application, holding that the applicants had failed to prove their claim of possession and ownership
of the land sought to be registered. 11 The applicants appealed to the respondent court, * which reversed the trial court and
recognized the claims of the applicant, but subject to the rights of Benguet and Atok respecting their mining claims. 12 In other
words, the Court of Appeals affirmed the surface rights of the de la Rosas over the land while at the same time reserving the
sub-surface rights of Benguet and Atok by virtue of their mining claims.

Both Benguet and Atok have appealed to this Court, invoking their superior right of ownership. The Republic has filed its own
petition for review and reiterates its argument that neither the private respondents nor the two mining companies have any
valid claim to the land because it is not alienable and registerable.

It is true that the subject property was considered forest land and included in the Central Cordillera Forest Reserve, but this did
not impair the rights already vested in Benguet and Atok at that time. The Court of Appeals correctly declared that:

There is no question that the 9 lots applied for are within the June Bug mineral claims of Benguet and the "Fredia and Emma"
mineral claims of Atok. The June Bug mineral claim of plaintiff Benguet was one of the 16 mining claims of James E. Kelly,
American and mining locator. He filed his declaration of the location of the June Bug mineral and the same was recorded in the
Mining Recorder's Office on October 14, 1909. All of the Kelly claims ha subsequently been acquired by Benguet Consolidated,
Inc. Benguet's evidence is that it had made improvements on the June Bug mineral claim consisting of mine tunnels prior to
1935. It had submitted the required affidavit of annual assessment. After World War II, Benguet introduced improvements on
mineral claim June Bug, and also conducted geological mappings, geological sampling and trench side cuts. In 1948, Benguet
redeclared the "June Bug" for taxation and had religiously paid the taxes.

The Emma and Fredia claims were two of the several claims of Harrison registered in 1931, and which Atok representatives
acquired. Portions of Lots 1 to 5 and all of Lots 6 to 9 are within the Emma and Fredia mineral claims of Atok Big Wedge Mining
Company.

The June Bug mineral claim of Benguet and the Fredia and Emma mineral claims of Atok having been perfected prior to the
approval of the Constitution of the Philippines of 1935, they were removed from the public domain and had become private
properties of Benguet and Atok.

It is not disputed that the location of the mining claim under consideration was perfected prior to November 15, 1935, when
the Government of the Commonwealth was inaugurated; and according to the laws existing at that time, as construed and
applied by this court in McDaniel v. Apacible and Cuisia (42 Phil. 749), a valid location of a mining claim segregated the area
from the public domain. Said the court in that case: The moment the locator discovered a valuable mineral deposit on the lands
located, and perfected his location in accordance with law, the power of the United States Government to deprive him of the
exclusive right to the possession and enjoyment of the located claim was gone, the lands had become mineral lands and they
were exempted from lands that could be granted to any other person. The reservations of public lands cannot be made so as to
include prior mineral perfected locations; and, of course, if a valid mining location is made upon public lands afterwards
included in a reservation, such inclusion or reservation does not affect the validity of the former location. By such location and
perfection, the land located is segregated from the public domain even as against the Government. (Union Oil Co. v. Smith, 249
U.S. 337; Van Mess v. Roonet, 160 Cal. 131; 27 Cyc. 546).

"The legal effect of a valid location of a mining claim is not only to segregate the area from the public domain, but to grant to
the locator the beneficial ownership of the claim and the right to a patent therefor upon compliance with the terms and
conditions prescribed by law. Where there is a valid location of a mining claim, the area becomes segregated from the public
domain and the property of the locator." (St. Louis Mining & Milling Co. v. Montana Mining Co., 171 U.S. 650; 655; 43 Law ed.,
320, 322.) "When a location of a mining claim is perfected it has the effect of a grant by the United States of the right of present
and exclusive possession, with the right to the exclusive enjoyment of all the surface ground as well as of all the minerals within
the lines of the claim, except as limited by the extralateral right of adjoining locators; and this is the locator's right before as
well as after the issuance of the patent. While a lode locator acquires a vested property right by virtue of his location made in
compliance with the mining laws, the fee remains in the government until patent issues."(18 R.C.L. 1152) (Gold Creek Mining
Corporation v. Hon. Eulogio Rodriguez, Sec. of Agriculture and Commerce, and Quirico Abadilla, Director of the Bureau of
Mines, 66 Phil. 259, 265-266)

It is of no importance whether Benguet and Atok had secured a patent for as held in the Gold Creek Mining Corp. Case, for all
physical purposes of ownership, the owner is not required to secure a patent as long as he complies with the provisions of the
mining laws; his possessory right, for all practical purposes of ownership, is as good as though secured by patent.

We agree likewise with the oppositors that having complied with all the requirements of the mining laws, the claims were
removed from the public domain, and not even the government of the Philippines can take away this right from them. The
reason is obvious. Having become the private properties of the oppositors, they cannot be deprived thereof without due
process of law. 13

Such rights were not affected either by the stricture in the Commonwealth Constitution against the alienation of all lands of the
public domain except those agricultural in nature for this was made subject to existing rights. Thus, in its Article XIII, Section 1, it
was categorically provided that:

SEC. 1. All agricultural, timber and mineral lands of the public domain, waters, minerals, coal, petroleum and other mineral oils,
all forces of potential energy and other natural resources of the Philipppines belong to the State, and their disposition,
exploitation, development, or utilization shall be limited to citizens of the Philippines or to corporations or associations at least
60% of the capital of which is owned by such citizens, subject to any existing right, grant, lease or concession at the time of the
inauguration of the government established under this Constitution. Natural resources with the exception of public agricultural
lands, shall not be alienated, and no license, concession, or lease for the exploitation, development or utilization of any of the
natural resources shall be granted for a period exceeding 25 years, except as to water rights for irrigation, water supply,
fisheries, or industrial uses other than the development of water power, in which case beneficial use may be the measure and
the limit of the grant.

Implementing this provision, Act No. 4268, approved on November 8, 1935, declared:

Any provision of existing laws, executive order, proclamation to the contrary notwithstanding, all locations of mining claim
made prior to February 8, 1935 within lands set apart as forest reserve under Sec. 1826 of the Revised Administrative Code
which would be valid and subsisting location except to the existence of said reserve are hereby declared to be valid and
subsisting locations as of the date of their respective locations.

The perfection of the mining claim converted the property to mineral land and under the laws then in force removed it from
the public domain. 14 By such act, the locators acquired exclusive rights over the land, against even the government, without
need of any further act such as the purchase of the land or the obtention of a patent over it. 15 As the land had become the
private property of the locators, they had the right to transfer the same, as they did, to Benguet and Atok.

It is true, as the Court of Appeals observed, that such private property was subject to the "vicissitudes of ownership," or even to
forfeiture by non-user or abandonment or, as the private respondents aver, by acquisitive prescription. However, the method
invoked by the de la Rosas is not available in the case at bar, for two reasons.

First, the trial court found that the evidence of open, continuous, adverse and exclusive possession submitted by the applicants
was insufficient to support their claim of ownership. They themselves had acquired the land only in 1964 and applied for its
registration in 1965, relying on the earlier alleged possession of their predecessors-in-interest. 16 The trial judge, who had the
opportunity to consider the evidence first-hand and observe the demeanor of the witnesses and test their credibility was not
convinced. We defer to his judgment in the absence of a showing that it was reached with grave abuse of discretion or without
sufficient basis. 17

Second, even if it be assumed that the predecessors-in-interest of the de la Rosas had really been in possession of the subject
property, their possession was not in the concept of owner of the mining claim but of the property as agricultural land, which it
was not. The property was mineral land, and they were claiming it as agricultural land. They were not disputing the lights of the
mining locators nor were they seeking to oust them as such and to replace them in the mining of the land. In fact, Balbalio
testified that she was aware of the diggings being undertaken "down below" 18 but she did not mind, much less protest, the
same although she claimed to be the owner of the said land.

The Court of Appeals justified this by saying there is "no conflict of interest" between the owners of the surface rights and the
owners of the sub-surface rights. This is rather doctrine, for it is a well-known principle that the owner of piece of land has
rights not only to its surface but also to everything underneath and the airspace above it up to a reasonable height. 19 Under
the aforesaid ruling, the land is classified as mineral underneath and agricultural on the surface, subject to separate claims of
title. This is also difficult to understand, especially in its practical application.

Under the theory of the respondent court, the surface owner will be planting on the land while the mining locator will be boring
tunnels underneath. The farmer cannot dig a well because he may interfere with the operations below and the miner cannot
blast a tunnel lest he destroy the crops above. How deep can the farmer, and how high can the miner, go without encroaching
on each other's rights? Where is the dividing line between the surface and the sub-surface rights?

The Court feels that the rights over the land are indivisible and that the land itself cannot be half agricultural and half mineral.
The classification must be categorical; the land must be either completely mineral or completely agricultural. In the instant
case, as already observed, the land which was originally classified as forest land ceased to be so and became mineral — and
completely mineral — once the mining claims were perfected. 20 As long as mining operations were being undertaken thereon,
or underneath, it did not cease to be so and become agricultural, even if only partly so, because it was enclosed with a fence
and was cultivated by those who were unlawfully occupying the surface.

What must have misled the respondent court is Commonwealth Act No. 137, providing as follows:

Sec. 3. All mineral lands of the public domain and minerals belong to the State, and their disposition, exploitation, development
or utilization, shall be limited to citizens of the Philippines, or to corporations, or associations, at least 60% of the capital of
which is owned by such citizens, subject to any existing right, grant, lease or concession at the time of the inauguration of
government established under the Constitution.

SEC. 4. The ownership of, and the right to the use of land for agricultural, industrial, commercial, residential, or for any purpose
other than mining does not include the ownership of, nor the right to extract or utilize, the minerals which may be found on or
under the surface.

SEC. 5. The ownership of, and the right to extract and utilize, the minerals included within all areas for which public agricultural
land patents are granted are excluded and excepted from all such patents.
SEC. 6. The ownership of, and the right to extract and utilize, the minerals included within all areas for which Torrens titles are
granted are excluded and excepted from all such titles.

This is an application of the Regalian doctrine which, as its name implies, is intended for the benefit of the State, not of private
persons. The rule simply reserves to the State all minerals that may be found in public and even private land devoted to
"agricultural, industrial, commercial, residential or (for) any purpose other than mining." Thus, if a person is the owner of
agricultural land in which minerals are discovered, his ownership of such land does not give him the right to extract or utilize
the said minerals without the permission of the State to which such minerals belong.

The flaw in the reasoning of the respondent court is in supposing that the rights over the land could be used for both mining
and non-mining purposes simultaneously. The correct interpretation is that once minerals are discovered in the land, whatever
the use to which it is being devoted at the time, such use may be discontinued by the State to enable it to extract the minerals
therein in the exercise of its sovereign prerogative. The land is thus converted to mineral land and may not be used by any
private party, including the registered owner thereof, for any other purpose that will impede the mining operations to be
undertaken therein, For the loss sustained by such owner, he is of course entitled to just compensation under the Mining Laws
or in appropriate expropriation proceedings. 21

Our holding is that Benguet and Atok have exclusive rights to the property in question by virtue of their respective mining
claims which they validly acquired before the Constitution of 1935 prohibited the alienation of all lands of the public domain
except agricultural lands, subject to vested rights existing at the time of its adoption. The land was not and could not have been
transferred to the private respondents by virtue of acquisitive prescription, nor could its use be shared simultaneously by them
and the mining companies for agricultural and mineral purposes.

WHEREFORE, the decision of the respondent court dated April 30, 1976, is SET ASIDE and that of the trial court dated March 11,
1969, is REINSTATED, without any pronouncement as to costs.

SO ORDERED.

Republic vs Leonor Dec. 23, 2009

DECISION

NACHURA, J.:

This is a petition for review on certiorari of the Court of Appeals (CA) Decision[1] dated December 19, 2003. The assailed
decision adjudged the cancellation of the free patents and original certificates of title (OCTs) over two of the five lots in
question in favor of petitioner.

The antecedents of the case are as follows:

On December 16, 1991, petitioner Republic of the Philippines, represented by the Regional Executive Director, Department of
Environment and Natural Resources (DENR), Region IV, through the Office of the Solicitor General, filed separate complaints for
Cancellation of Free Patent and OCT and Reversion against respondents Ignacio Leonor and Catalino Razon. The complaints
involved the following properties:

1. In Civil Case No. 55-91: Free Patent No. (IV-3A)-2182, covered by OCT No. P-1676 in the name of Ignacio Leonor, over
Lot No. 10108, Cad. 511, Lemery Cadastre with an area of 722 square meters;
2. In Civil Case No. 56-91: Free Patent No. (IV-3A)-2181, covered by OCT No. P-1675 in the name of Ignacio Leonor,
over Lot No. 8617, Cad. 511, Lemery Cadastre with an area of 706 square meters;

3. In Civil Case No. 57-91: Free Patent No. (IV-3A)-2180, covered by OCT No. P-1674 in the name of Catalino Razon, over
Lot No.10109, Cad. 511, Lemery Cadastre, with an area of 722 square meters;

4. In Civil Case No. 58-91: Free Patent No. (IV-3A)-1891, covered by OCT No. P-1127 in the name of Ignacio Leonor,
over Lot No. 9398, Cad. 511, Lemery Cadastre with an area of 2,066 square meters;

5. In Civil Case No. 59-91: Free Patent No. (IV-3A)-1892, covered by OCT No. P-1128 in the name of Catalino Razon,
over Lot No. 9675, Cad. 511, Lemery Cadastre with an area of 1,944 square meters.[2]

In Civil Case Nos. 55-91,[3] 56-91[4] and 57-91,[5] the complaints averred that, in an investigation conducted by DENR-Region IV, it
was ascertained that Lot Nos. 10108, 8617 and 10109 were part of the non-disposable foreshore land and did not appear in the
cadastral map or in the cadastral records as having been officially surveyed by the DENR. These defects allegedly constituted
fraud which, in effect, ipso facto cancelled the free patents and the corresponding OCTs.

In contrast, the complaints in Civil Case Nos. 58-91[6] and 59-91[7] alleged that, on the basis of a protest filed by Luisa Ilagan Vda.
de Agoncillo who claimed to be in possession of Lot Nos. 9398 and 9675 since time immemorial, an investigation was
conducted by the DENR wherein it was discovered that (1) although the said lots appeared in the cadastral map, they were not
cadastrally surveyed or approved cadastral lots as evidenced by the Alphabetical and Numerical List of Claimants; (2) the lots
were verified to be part of the early survey conducted on June 22, 1977 and identified as Lot No. 6192 of Cadastre 511, Lemery
Cadastre, subsequently covered by Plan SWO-4A-000306-D in the name of Luisa Ilagan; and (3) Lot Nos. 9398 and 9675 were
conveyed to respondents, respectively, through an Affidavit of Relinquishment of Rights executed on November 27, 1986 by a
certain Anacleto Serwelas who had no right whatsoever over the land. The complaints further averred that serious
discrepancies existed among the technical descriptions appearing in the certificates of title, the cadastral map and the transfer
of rights. These defects, according to the complaint, also constituted fraud which, in effect,ipso facto cancelled the said patents
and the corresponding OCTs.

On February 10, 1992, respondents filed their separate answers[8] uniformly stating as follows: (1) the free patents were issued
in accordance with existing law and procedure; (2) the subject lots were surveyed by Geodetic Engineer Alexander Jacob of the
Bureau of Lands and inspected and certified to be alienable and disposable by the Land Inspector of the Bureau of Lands; (3)
the right of action for the cancellation of the same had already prescribed since more than one year had already lapsed since
the free patents were issued; (4) they had been in continuous, exclusive and notorious possession and occupation of the lots for
more than 30 years and they had developed them into a beach resort, with valuable facilities; and (5) the subject lots were not
investigated by the DENR-Region IV and there was no resolution issued by the said office to that effect.

Luisa Ilagan was allowed to intervene in Civil Case Nos. 58-91 and 59-91. She claimed that Lot Nos. 9398 and 9675 were part of
the parcel of land that she owned, designated as Lot No. 6192, Cad-511-D of the Lemery Cadastre and covered by Tax
Declaration No. 0527; that this parcel of land was surveyed on June 22, 1977 and Plan SWO-4A-000306-D was approved on
April 18, 1980; that she had been in peaceful possession of the subject land for more than 60 years but, because of old age, she
failed to visit and supervise the land; that Anacleto Serwelas was her tenant who took advantage of her absence and succeeded
in selling the western portion of the subject land in favor of respondents, without her knowledge and consent; and that in 1987,
she learned of respondents applications for free patent and of the issuance of the OCTs in their names; hence, she filed a
formal protest with the DENR asking for an investigation.[9]

In answer to these allegations, respondents averred that Luisa Ilagan had already sold her properties to her tenants, and that
Plan SWO-4A-000304 in her name was rejected by the Bureau of Lands as shown in the Cadastral Map of Lemery Cadastre, Cad.
511, Case 22.[10] Luisa Ilagan replied that the rejection of Plan SWO-4A-000304 was null and void for lack of notice. She insisted
that respondents had no right over the subject lots since they acquired them from Anacleto Serwelas, who was not the owner
of the properties.
On June 14, 2000, the Regional Trial Court rendered a decision in favor of respondents, thus:

WHEREFORE, for insufficiency of evidence presented by the plaintiff Republic of the Philippines and the Intervenor, to prove
that fraud was committed to acquire the title of the land in dispute, all the above five entitled cases are hereby ordered
DISMISSED for lack of merit.

IT IS SO ORDERED.[11]

The heirs of Luisa Ilagan and the petitioner filed separate appeals with the CA.

On February 11, 2002, the CA partially granted petitioners prayers. It declared that two of the five lotsLot Nos. 10108 and
10109are foreshore lands. The CA noted that (a) serious discrepancies exist between the cadastral map and the technical
description in the OCTs covering these two lots; (b) the said lots do not appear in the cadastral map; (c) Atty. Raymundo L.
Apuhin, petitioners witness, testified that the said lots were not surveyed and approved by the DENR; and (d) they do not
appear to be covered by corresponding tax declarations. Based on the foregoing, the CA concluded that these two lots are
foreshore lands. Consequently, it ordered the cancellation of Free Patent No. (IV-3A)-2182 and OCT No. P-1676 over Lot No.
10108 and Free Patent No. (IV-3A)-2180 and OCT No. P-1674 over Lot No. 10109. As for Lot Nos. 8617, 9398 and 9675, the CA
sustained the trial courts finding that there was no sufficient evidence to prove that they are foreshore lands or part of Luisa
Ilagans property. The dispositive portion of the decision reads:

WHEREFORE, premises considered, the assailed decision dated June 14, 2000 of the RTC, Branch 5, Lemery, Batangas in Civil
Cases Nos. 55-91 to 59-91 is hereby AFFIRMED with MODIFICATION. Free Patent No. (IV-3A)-2182 with the corresponding OCT
No. P-1676 in the name of Ignacio Leonor over Lot No. 10108, and Free Patent No. (IV-3A)-2180 with the corresponding OCT
No. P-1674, in the name of Catalino Razon over Lot [No.] 10109 are hereby ordered CANCELLED from the Registry of Deeds of
Batangas.

The rest of the decision stands.

SO ORDERED.[12]

This petition for review on certiorari seeks the reversion of Lot Nos. 8617, 9398 and 9675 to petitioner. On this score, petitioner
ascribes the following error to the appellate court:

THE COURT OF APPEALS COMMITTED SERIOUS ERROR OF LAW WHEN IT SUSTAINED THE VALIDITY OF THE THREE SUBJECT FREE
PATENTS AND TITLES ALBEIT THEY PERTAIN TO INALIENABLE FORESHORE LANDS AND DESPITE THE FRAUDULENT ENTRIES IN
RESPONDENTS FREE PATENT APPLICATIONS.[13]

Petitioner argues that the lands are inalienable foreshore lands. It points out that the five lots comprise the whole Leonor
Beach Resort and that when the technical descriptions of the subject lots were plotted on the cadastral map of Barangay
Nonong Castro, the lots were identified as foreshore lands, which are not capable of appropriation. [14] Petitioner adds that the
burden is on respondents to prove that the lands that have been registered in their names are alienable and disposable.[15]

Petitioner further contends that, assuming that the subject lands are not foreshore lands, the free patents should nonetheless
be cancelled, because respondents committed fraud and made misrepresentations in their free patent applications in that (a)
they declared that the subject lots were cadastrally surveyed when, in truth, they do not appear in the approved Cadastral Plan
of Lemery, Batangas, Cad. 511, Case 22; (b) respondent Ignacio Leonor declared that he acquired Lot No. 9398 from Moises and
Ricardo Peren and Vicente de Roxas, whose names do not however appear on the lists of claimants for Barangay Nonong
Castro, Case 22, Lemery Cadastre, indicating that they are fictitious persons; (c) respondent Ignacio Leonor failed to enter the
names of his predecessors-in-interest as to Lot No. 8617, as required in the free patent application; (d) serious discrepancies
were noted in the description of Lot No. 9398 in the application for free patent and in the technical description in OCT No. P-
1127; and (e) Lot No. 9675 does not appear in the lists of claimants.[16]
Incidentally, it should be pointed out that, other than Lot Nos. 10108 and 10109, only Lot No. 8617 was alleged in the complaint
(Civil Case No. 59-91) to be part of the indisposable foreshore land. In fact, there is no piece of evidence pointing to Lot Nos.
9398 and 9675 as being foreshore lands. Petitioner seeks the cancellation of the free patents over Lot Nos. 9398 and 9675
solely on the ground that they were procured through fraud and misrepresentation.

The Court finds that the petition has no merit

As a rule, the findings of fact of the trial court when affirmed by the CA are final and conclusive on, and cannot be reviewed on
appeal by, this Court as long as they are borne out by the records or are based on substantial evidence. The Court is not a trier
of facts, its jurisdiction being limited to reviewing only errors of law that may have been committed by the lower courts. [17] But
to appease any doubt on the correctness of the assailed ruling, we have carefully perused the records and, nonetheless, arrived
at the same conclusion.

To be sure, petitioner was not able to adequately establish that Lot No. 8617 is a foreshore land or that the free patents
covering Lot Nos. 8617, 9398 and 9675 were procured through fraud or misrepresentation.

At the outset, petitioner argues that the burden to prove that the lands in question are alienable and disposable is upon
respondents. The argument is out of line. This case is not a land registration proceeding but involves reversion of lands already
registered in the names of respondents. At this stage, it would be reasonable to presume that respondents had established that
the properties are alienable and disposable considering that they have already succeeded in obtaining free patents and OCTs
over the properties. In this reversion proceeding, premised on the claim that the property is foreshore land or that the patents
were obtained through fraud or misrepresentation, the burden is now upon petitioner to prove such allegations.

With regard to Lot No. 8617, the records reveal that the only piece of evidence alluding to this lot being foreshore land is the
testimony of Atty. Apuhin of the DENR-Region IV, which is quoted as follows:

Q- And what did you find in relation to the Free Patent No. (IV-3A) 2180 insofar as that plotting made by the Legal
Division of the DENR is concerned?

A- In my request, I found out that x x x Lot No. 8617 is a foreshore lot.

Q- How about the survey record, what did you find insofar as Lot No. 8617 is concerned?

A- In verification with the Survey Division, Lot No. 8617 is definitely a part of [the] foreshore lot as shown in the
approved cadastral map of Lemery.

Q- By the way, when you speak of foreshore lot, what do you mean?

A- It is an area covered by the flow of tide in its highest equational tide which is 20 meters from the highest equational
tide.

Q- If it is a foreshore land, can it be the subject of Free Patent application?

A- No, sir.

xxxx
Q- After conducting the necessary investigation insofar as Lot 8617 is concerned, what is your conclusion?

A- After conducting the necessary investigation insofar as Lot 8617 is concerned, Lot 8617 is a foreshore lot.

Q- If it is a foreshore lot, what is your conclusion?

A- A foreshore lot cannot be the subject of acquisition [of] Free Patent.

Q- If it cannot be the subject of acquisition, what is the effect on the application for Free Patent of x x Lot No. 8617?

A- The application for x x x Free Patent should not have been approved.[18]

Certainly, Atty. Apuhins testimony fails to convince us. The interview markedly lacks details as to how he conducted an
investigation to determine whether Lot No. 8617 is foreshore land or an explanation as to how he arrived at his
conclusion. Although it was stated in the records that Atty. Apuhin conducted an ocular inspection, his only finding on the basis
of this inspection was that the lots had already been developed as a beach resort. In his direct testimony, he vaguely stated that
the lot is foreshore land as shown in the cadastral map. We have examined the said cadastral map of Barangay Nonong Castro,
Lemery, Batangas but we noticed that it does not indicate, in any way, that Lot No. 8617 is foreshore land. What is obvious in
the said map is that the lot is close to the waters of Balayan Bay. However, the lands proximity alone to the waters alone does
not necessarily make it a foreshore land.[19] It must be shown that the land is between high and low water and left dry by the
flux and reflux of the tides or between the high and low water marks, which is alternatively wet and dry according to the flow of
the tide.[20]

We likewise agree with the CA that petitioner was not able to establish that fraud or misrepresentation attended the
application for free patents. In the same way that petitioner has the burden of proving that Lot No. 8617 is a foreshore land,
petitioner, as the party alleging that fraud and misrepresentation vitiated the application for free patents, also bears the
burden of proof.[21] Fraud and misrepresentation are never presumed, but must be proved by clear and convincing evidence;
mere preponderance of evidence is not even adequate.[22]

To show that there was fraud, petitioner insists that the three lots do not appear in the cadastral map of Barangay Nonong
Castro, which allegedly indicates that they were not cadastrally surveyed. This is manifestly untrue as the said cadastral map of
Lemery Cadastre, Cad. 511, Case 22, clearly includes and indicates the locations of Lot Nos. 8617, 9398 and 9675.

Petitioner also cites discrepancies in the description of Lot No. 9398 in the free patent application and in the technical
description in OCT No. P-1127. If true, such discrepancies would not necessarily imply that respondents employed fraud or
misrepresentation in obtaining the free patent. After all, there was no proof that the lot referred to in the free patent
application was different from the lot described in OCT No. P-1127.

Further, petitioner points out that in the free patent application for Lot No. 8617, respondent Ignacio Leonor did not indicate
the names of his predecessors-in-interest and the date when he began his possession and cultivation of the lot. Petitioner
contends that this was in violation of Section 91[23] of the Public Land Act and, as such, resulted in the ipso facto cancellation of
the free patent.

The mere omission of an information from the patent application, though essential, does not, per se, cause the ipso
facto cancellation of the patent. It must be shown that the information withheld would have resulted in the disapproval of the
free patent application had it been disclosed. The names of the predecessors-in-interest are obviously required to be indicated
in the application form in order to show that the applicant has complied with the occupation and cultivation requirement under
the law. In this case, petitioner had no evidence showing that respondents had not complied with the occupation and
cultivation requirement under the law. Considering this, we are ill-equipped to pronounce the ipso facto cancellation of free
patents.
With regard to Lot No. 9398, petitioner argues that the names of the predecessors of Ignacio Leonor Vicente de Roxas, Moises
and Ricardo Peren listed in the application form for the free patent are fictitious persons, as their names do not appear in the
List of Claimants. The list also does not include Lot No. 9675, which petitioner claims is an indication that the lot was not
cadastrally surveyed.

Again, we are not convinced. Undoubtedly, the list of claimants is evidence that the lots enumerated therein were cadastrally
surveyed, and that the name indicated after each lot number was that of the claimant of the lot at the time of the survey. But
despite Atty. Apuhins testimony, the list cannot be given weight particularly with respect to lots not included therein. In other
words, the list cannot be taken as evidence that lots that were not included in the list were not cadastrally surveyed or that only
the claimants named therein had rights over that particular lot. This is only reasonable considering that it is not even known, for
sure, when the list was made, how it was prepared, and how often it was updated.

Atty. Apuhins testimony on the preparation of the list and on there being no other list for other lots in Barangay Nonong Castro
is not worthy of credence. He admitted during trial that he was not privy to the preparation of the list. Apparently, he was also
not the actual custodian of the list since a certain Florencio V. Carreon, Chief, Records Unit, certified the copy of the list. Atty.
Apuhins ignorance on this matter is made more apparent by the following testimony:

Q- You were a privy in the preparation of the list?

A- I am not.

Q- Since you were not a privy to the preparation of the list[,] you must have inquired how often was the list
prepared[.] The list given to you was the list on the approved Cadastral Survey?

A- Case No. 22.

Court:

Q- The question of the Court is that, constantly this list is revised because of the approval of certain claimants in
relation to the approved survey?

A- Yes, sir.

Q- As you inquired, what was the date when this list shown to you alphabetically was approved?

A- It was approved on March 12, 1987.

Q- Because the alphabetical list is based on that?

A- Yes, sir.

xxxx

Q- Do you know, based on record when the cadastral survey was implemented, that cadastral survey which was
approved on March 12, 1987?

A- I did not see.

Q- This list of claimants, they are listed while the cadastral survey is being done?

A- Yes, sir.

Q- Based on the approved survey plan?


A- Yes, sir.

Q- It would appear at the time of the survey?

A- Yes, sir.

Q- The survey plan is approved later on?

A- Yes, sir.

Q- On your own knowledge based on the investigation, was there any list subsequent to March 12, 1987 released by the DENR?

A- None, sir.

Q- But you look[ed] at this application on your investigation?

A- Yes, sir.

Q- And you affirm that this was the last list of claimants alphabetically done in relation to [Barangay] Nonong Castro?

A- Yes, sir.

Fiscal:

Q- Aside from this list of claimants, has there been other list of claimants, prior list?

A- I have no knowledge[24]

We also do not believe that Moises and Ricardo Peren and Vicente de Roxas are fictitious persons. From the investigation
conducted by Atty. Apuhin, he learned that Moises Peren executed a Waiver of Real Rights on June 16, 1986 in favor of Ignacio
Leonor.[25] It also appears from the records that these persons were respondents in a case for accion reivindicatoria and
quieting of title filed by Luisa Ilagan.[26]

From the foregoing, the dearth of petitioners evidence is glaring. DENR-Region IV did not conduct a thorough investigation of
the alleged irregularities imputed to respondents in obtaining the free patents. There was not even a written report on the
investigation submitted to the court. In view of this, we are constrained to sustain the findings of both the trial court and the
appellate court and to deny the petition.

WHEREFORE, premises considered, the petition is DENIED. The Court of Appeals Decision dated December 19,
2003 is AFFIRMED.

Almagro vs Kwan Oct. 20, 2010

This is a consolidation of two separate petitions for review,[1] assailing the 4 April 2006 Decision[2] and the 31 October 2006
Resolution[3] of the Court of Appeals in CA-G.R. SP Nos. 71237 and 71437.

This case involves Lot No. 6278-M, a 17,181 square meter parcel of land covered by TCT No. T-11397. Lot No. 6278-M is located
at Maslog, Sibulan, Negros Oriental and is registered in the name of spouses Kwan Chin and Zosima Sarana. Respondents are
the legitimate children of spouses Kwan Chin and Zosima Sarana, who both died intestate on 2 November 1986 and 23 January
1976, respectively, in Dumaguete City. Upon the death of their parents, respondents inherited Lot No. 6278-M through
hereditary succession.
On 18 September 1996, respondents filed with the Municipal Trial Court (MTC) an action for recovery of possession and
damages against spouses Rogelio and Lourdes Duran, spouses Romulo Vinalver and Elsa Vinalver,[4] spouses Marte[5] Bati-on
and Liz E. Bati-on, spouses Pablo Deciar and Marlyn Deciar, spouses Salvador Palongpalong and Bienvenida Palongpalong,
spouses Sabas Kiskis and Eduvigis Kiskis, spouses Pio Tubat, Jr. and Encarnita Tubat, spouses Andres Tubat and Leonides Tubat,
spouses George Tubat and Noela Tubat, spouses Dodong Go and Alice Go, spouses Delano Bangay and Maria Bangay, [6] spouses
Simeon Pachoro and Margarita Pachoro, spouses Cepriano[7]Tubat and Elsa Tubat, spouses Jovito Remolano and Editha Orlina
Remolano, spouses Nelson Miravalles and Erlene Miravalles, Dronica Orlina,[8] Clarita Barot Lara, Conchita Orlina, Antonia
Malahay and the Philippine National Police (PNP),[9] Agan-an, Sibulan, Negros Oriental. Subsequently, spouses Manuel Almagro
and Elizabeth Almagro intervened as successors-in-interest of spouses Delano Bangay and Maria Bangay.

During pre-trial, the parties agreed to refer the case to the Chief of the Land Management Services Division, PENRO-DENR,
Dumaguete City, to conduct a verification survey ofLot No. 6278-M. When the PENRO personnel failed to conduct the
verification survey, the court and the parties designated Geodetic Engineer Jorge Suasin, Sr. (Engr. Suasin) as joint
commissioner to do the task. Engr. Suasin conducted the verification and relocation survey of Lot No. 6278-M on 12-13
September 2000 in the presence of the parties, some of their lawyers, and the MTC Clerk of Court. Thereafter, Engr. Suasin
submitted a written report with the following findings:

WRITTEN REPORT

Comes now, the undersigned Geodetic Engineer Jorge S. Suasin, Sr., to this Honorable Court, most respectfully submit the
following written report of the verification and relocation survey of the lot 6278-M located at Maslog, Sibulan, Negros Oriental
with T.C.T. No. T-11397 owned by Salvacion G. Kwan, et al.

A. That a big portion of the lot is submerged under the sea and only a small portion remain as dry land.

B. That some of the defendants have constructed their buildings or houses inside the dry land while others have constructed
outside or only a small portion of their buildings or houses are on the said dry land.

The defendants and their buildings or houses are as follows:

1. Sps. Rogelio Duran . . . . . . . . . . . . . . . . . . . . . . . . . inside

2. Sps. Romulo Vinalver. . . . . . . . . . . . . . . . . . . . . . . inside

3. Sps. Marto Bati-on . . . . . . . . . . . . . . . . . . . . . . . . . inside

4. Sps. Salvador Palongpalong . . . . . . . . . . . . . . . . . . inside

5. Sps. Pablo Deciar . . . . . . . . . . . . . . . . . . . . . . . . . . inside


6. Sps. Sabas Kiskis . . . . . . . . . . . . . . . . . . . . . . . . . . .inside

7. Sps. Pio Tubat, Jr. . . . . . . . . . . . . . . . . . . . . . . . 2 houses, the first house a portion, and the second one - inside

8. Sps. Andres Tubat . . . . . . . . . . . . . . . . . . . . . . . . . . inside

9. Sps. George Tubat . . . . . . . . . . . . . . . . . . . . . . . . . . portion

10. Sps. Dodong Go . . . . . . . . . . . . . . . . . . . . . . . . . . inside

11. Sps. Delano Bangay-Almagro . . . . . . . . . . . . . . . . portion

12. Sps. Simeon Pachoro . . . . . . . . . . . . . . . . . . . . . . . inside

13. Sps. Cipriano Tubat . . . . . . . . . . . . . . . . . . . . . . . . inside

14. Sps. Jovito Remolano . .. . . . . . . . . . . . . . . . . . . . . inside

15. Sps. Nelson Miravalles . . . . . . . . . . . . . . . . . . . . . cottage and

house - outside

16. Monica Orlina . . . . . . . . . . . . . . . . . . . . . . . . . . . . cottage inside

and house -

portion

17. Clarita Barot . . . . . . . . . . . . . . . . . . . . . . . . . . . . . . outside

18. Conchita Orlina . . . . . . . . . . . . . . . . . . . . . . . . . . . outside

19. Antonia Malahay . . . . . . . . . . . . . . . . . . . . . . . . . . outside

The verification and relocation survey was executed last September 12-13, 2000 with the presence of both parties and of the
Clerk of Court. The cost of the survey was FIFTEEN THOUSAND PESOS (P15,000) shouldered by the plaintiffs and the defendants
equally.

Enclosed are a blue print of the sketch plan and a xerox copy of the land title of the said lot.

Respectfully submitted by:

(Sgd) JORGE SUASIN, SR.

Geodetic Engineer

After the court admitted Engr. Suasin's report and the pleadings of the parties, respondents filed a motion for judgment on the
pleadings, which the MTC granted.
In its Judgment dated 11 May 2001, the MTC dismissed the complaint on the ground that the remaining dry portion of Lot No.
6278-M has become foreshore land and should be returned to the public domain. The MTC explained:

The term foreshore refers to that part of the land adjacent to the sea which is alternately covered and left dry by the ordinary
flow of the tides. Foreshore lands refers to the strip of land that lies between the high and low water marks and that is
alternately wet and dry according to the flow of the tide. The term foreshore land clearly does not include submerged lands.

From these definitions, it is safe to conclude that the remaining dry portion of Lot No. 6278-M is now foreshore land. A big
portion of the said lot is presently underwater or submerged under the sea. When the sea moves towards the estate and the
tide invades it, the invaded property becomes foreshore land and passes to the realm of public domain. The subject land, being
foreshore land, should therefore be returned to the public domain. Besides, Article 420 of the Civil Code provides:

Art. 420. The following thin[g]s are property of public dominion:

(1) Those intended for public use, such as roads, canals, rivers, torrents, ports and bridges constructed by the State, banks,
shores, roadsteads, and others of similar character;

Plaintiff cannot use the doctrine of indefeasibility of their Torrens title, as property in question is clearly foreshore land. At the
time of its registration, property was along the shores. In fact, it is bounded by the Taon Strait on the NW along lines 2-3-4. The
property was of public dominion and should not have been subject of registration. The survey showed that the sea had
advanced and the waves permanently invaded a big portion of the property making the land part of the shore or the beach. The
remaining dry land is foreshore and therefore should be returned to the public domain.[11]

Respondents appealed to the Regional Trial Court (RTC). The RTC conducted ocular inspections of Lot No. 6278-M on two
separate dates: on 5 October 2001 during low tide and on 15 October 2001 when the high tide registered 1.5 meters. All the
parties and their lawyers were notified before the two ocular inspections were conducted. During the ocular inspections, in
which some parties and their lawyers were present, the RTC observed that the small portion referred to by Engr. Suasin as dry
land in his report actually remained dry even during high tide.[12] Thus, the RTC concluded that the disputed remaining portion
of Lot No. 6278-M is not foreshore land. The RTC stated:

It is the Court's considered view that the small portion of plaintiff's property which remains as dry land is not within the scope
of the well-settled definition of foreshore and foreshore land as mentioned above. For one thing, the small dry portion is not
adjacent to the sea as the term adjacent as defined in Webster's Dictionary means contiguous or touching one another or lying
next to. Secondly, the small dry portion is not alternately wet and dry by the ordinary flow of the tides as it is dry land.
Granting, as posited by defendants, that at certain times of the year, said dry portion is reached by the waves, then that is not
anymore caused by the ordinary flow of the tide as contemplated in the above definition. The Court then finds that the
testimony of Engr. Suasin dovetails with the import and meaning of foreshore and foreshore land as defined above.

Anent the case of Republic vs. Court of Appeals, 281 SCRA 639, also cited in the appealed judgment, the same has a different
factual milieu. Said case involves a holder of a free patent on a parcel of land situated at Pinagtalleran, Caluag, Quezon who
mortgaged and leased portions thereof within the prescribed five-year period from the date of issuance of the patent. It was
established in said case that the land subject of the free patent is five (5) to six (6) feet deep under water during high tide and
two (2) feet deep at low tide. Such is not the situation of the remaining small dry portion which plaintiffs seek to recover in the
case at bar.[13]

On 8 January 2002, the RTC rendered its Decision,[14] the dispositive portion of which reads:

WHEREFORE, all told and circumspectly considered, the appealed judgment is hereby reversed and set aside insofar as it states
that plaintiffs are not entitled to recover possession of the property in question.
Plaintiffs-appellants have the right to recover possession of the remaining small dry portion of the subject property in question.
It is further ordered to remand this case to the court of origin for the reception of further evidence to determine who among
the defendants-appellees are builders or possessors in good faith and who are not and once determined, to apply accordingly
the pertinent laws and jurisprudence on the matter.

SO ORDERED.[15]

Petitioners moved for reconsideration, which the RTC denied in its Order[16] dated 6 May 2002.

Petitioners filed separate petitions for review with the Court of Appeals, alleging that the disputed portion of Lot No. 6278-M is
no longer private land but has become foreshore land and is now part of the public domain.

The Ruling of the Court of Appeals

On 4 April 2006, the Court of Appeals promulgated its decision, affirming with modification the RTC Decision. The dispositive
portion of the Court of Appeals Decision[17]reads:

WHEREFORE, the instant petitions for review are DENIED. And the Decision dated January 8, 2002 of Branch 38 of the Regional
Trial Court of Dumaguete City is hereby AFFIRMED with MODIFICATION as regards the dispositive portion only. Based on the
written report of Geodetic Engr. Suasin categorically indentifying who among herein petitioners are illegally occupying a portion
of Lot No. 6278-M, the following petitioners are ordered to vacate the premises and/or remove the houses and/or cottages
constructed on Lot No. 6278-M within thirty (30) days from finality of judgment, namely: 1)Sps. Rogelio Duran, 2) Sps. Romulo
Vinalver, 3) Sps. Marto Bati-on, 4) Sps. Salvador Palongpalong, 5) Sps. Pablo Deciar, 6) Sps. Sabas Kiskis, 7) Sps. Pio Tubat, Jr.
(first house portion, second house inside), 8) Sps. Andres Tubat, 9) George Tubat (portion), 10) Sps. Dodong Go, 11) Sps. Delano
Bangay-Almagro (portion), 12) Sps. Simeon Pachoro, 13) Sps. Cipriano Tubat, 14) Sps. Jovito Remolano and 15) Monica Orlina
(cottageinside and house portion).

Costs against petitioners.

SO ORDERED.

In modifying the RTC Decision, the Court of Appeals explained:

Lastly, the argument that the RTC decision was vague and indefinite is utterly bereft of merit. We have found no reversible
error in the appreciation of the facts and in the application of the law by the RTC which will warrant the reversal of the
questioned decision. However, litigation must end and terminate sometime and somewhere, and it is essential to the
administration of justice that the issues or causes therein should be laid to rest. Hence, in keeping with this principle, We
modify the assailed decision insofar as the dispositive portion is concerned. It is our considered view that there is no longer a
need to determine who among the petitioners are builders in good faith or not considering that it has been established in the
MTC that they knew all along that the subject lot is a titled property. As such, petitioners should vacate and/or demolish the
houses and/or cottages they constructed on Lot No. 6278-M as stated in the written report of Geodetic Engineer Jorge S.
Suasin, Sr. Remanding this case to the court of origin would not only unduly prolong the resolution of the issues of this case, but
would also subject the parties to unnecessary expenses.[19]

Hence, these consolidated petitions.

The Issue
The primary issue in this case is whether the disputed portion of Lot No. 6278-M is still private land or has become foreshore
land which forms part of the public domain

The Ruling of the Court

We find the petitions without merit.

Petitioners contend that the disputed portion of Lot No. 6278-M is already foreshore land. In fact, most of them allegedly have
foreshore lease permits from the Department of Environment and Natural Resources (DENR) on the said foreshore land.

However, petitioners failed to present evidence to prove their claim that they are holders of foreshore lease permits from the
DENR. Thus, the RTC Order dated 6 May 2002 stated:

Defendants-appellees have been harping that they have been granted foreshore leases by DENR. However, this is merely lip
service and not supported at all by concrete evidence. Not even an iota of evidence was submitted to the lower court to show
that defendants-appellees herein have been granted foreshore leases.[20]

Although the MTC concluded that the subject land is foreshore land, we find such conclusion contrary to the evidence on
record.

It is undisputed that the subject land is part of Lot No. 6278-M, which is covered by TCT No. T-11397, registered in the name of
respondents' parents, Kwan Chin and Zosimo Sarana. In fact, as found by the Court of Appeals, even the Provincial Environment
and Natural Resources Officer (PENRO) declared in May 1996 that Lot No. 6278-M is a private property covered by a Torrens
Title and that petitioners should vacate the disputed property or make other arrangements with respondents. [21]

Furthermore, from the report of Engr. Suasin, the geodetic engineer designated by the court and the parties as joint
commissioner to conduct the survey, it can be clearly gleaned that the contested land is the small portion of dry land of Lot No.
6278-M. Even in his testimony, Engr. Suasin was adamant in stating that the remaining portion of Lot No. 6278-M is not
foreshore because it is already dry land and is away from the shoreline.[22] Because of this apparent contradiction between the
evidence and the conclusion of the MTC, the RTC conducted ocular inspection twice, during low tide and high tide, and
observed that the disputed portion of Lot No. 6278-M actually remained dry land even during high tide. Thus, the RTC
concluded that the said land is not foreshore land. On appeal, the Court of Appeals adopted the findings and conclusion of the
RTC that the disputed land is not foreshore land and that it remains as private land owned by respondents.

We are in accord with the conclusion of the Court of Appeals and the RTC that the disputed land is not foreshore land. To
qualify as foreshore land, it must be shown that the land lies between the high and low water marks and is alternately wet and
dry according to the flow of the tide.[23] The land's proximity to the waters alone does not automatically make it a foreshore
land.[24]

Thus, in Republic of the Philippines v. Lensico,[25] the Court held that although the two corners of the subject lot adjoins the sea,
the lot cannot be considered as foreshore land since it has not been proven that the lot was covered by water during high tide.

Similarly in this case, it was clearly proven that the disputed land remained dry even during high tide. Indeed, all the evidence
supports the conclusion that the disputed portion of Lot No. 6278-M is not foreshore land but remains private land owned by
respondents.

WHEREFORE, we DENY the petitions. We AFFIRM the 4 April 2006 Decision and the 31 October 2006 Resolution of the Court of
Appeals in CA-G.R. SP Nos. 71237 and 71437.

office of the city mayor of paranaque vs ebio June 23, 2010


VILLARAMA, JR., J.:

Before us is a petition for review on certiorari under Rule 45 of the 1997 Rules of Civil Procedure, as amended, assailing the
January 31, 2007 Decision[1] and June 8, 2007 Resolution[2] of the Court of Appeals (CA) in CA-G.R. SP No. 91350 allegedly for
being contrary to law and jurisprudence. The CA had reversed the Order[3] of the Regional Trial Court (RTC) of Paraaque City,
Branch 196, issued on April 29, 2005 in Civil Case No. 05-0155.

Below are the facts.

Respondents claim that they are the absolute owners of a parcel of land consisting of 406 square meters, more or less, located
at 9781 Vitalez Compound in Barangay Vitalez, Paraaque City and covered by Tax Declaration Nos. 01027 and 01472 in the
name of respondent Mario D. Ebio. Said land was an accretion of Cut-cut creek. Respondents assert that the original occupant
and possessor of the said parcel of land was their great grandfather, Jose Vitalez. Sometime in 1930, Jose gave the land to his
son, Pedro Vitalez. From then on, Pedro continuously and exclusively occupied and possessed the said lot. In 1966, after
executing an affidavit declaring possession and occupancy,[4] Pedro was able to obtain a tax declaration over the said property
in his name.[5] Since then, respondents have been religiously paying real property taxes for the said property.[6]

Meanwhile, in 1961, respondent Mario Ebio married Pedros daughter, Zenaida. Upon Pedros advice, the couple established
their home on the said lot. In April 1964 and in October 1971, Mario Ebio secured building permits from the Paraaque municipal
office for the construction of their house within the said compound.[7] On April 21, 1987, Pedro executed a notarized Transfer
of Rights[8] ceding his claim over the entire parcel of land in favor of Mario Ebio. Subsequently, the tax declarations under
Pedros name were cancelled and new ones were issued in Mario Ebios name.[9]

On March 30, 1999, the Office of the Sangguniang Barangay of Vitalez passed Resolution No. 08, series of 1999[10] seeking
assistance from the City Government of Paraaque for the construction of an access road along Cut-cut Creek located in the said
barangay. The proposed road, projected to be eight (8) meters wide and sixty (60) meters long, will run from Urma Drive to the
main road of Vitalez Compound[11] traversing the lot occupied by the respondents. When the city government advised all the
affected residents to vacate the said area, respondents immediately registered their opposition thereto. As a result, the road
project was temporarily suspended.[12]

In January 2003, however, respondents were surprised when several officials from the barangay and the city planning office
proceeded to cut eight (8) coconut trees planted on the said lot. Respondents filed letter-complaints before the Regional
Director of the Bureau of Lands, the Department of Interior and Local Government and the Office of the Vice Mayor.[13] On
June 29, 2003, the Sangguniang Barangay of Vitalez held a meeting to discuss the construction of the proposed road. In the said
meeting, respondents asserted their opposition to the proposed project and their claim of ownership over the affected
property.[14] On November 14, 2003, respondents attended another meeting with officials from the city government, but no
definite agreement was reached by and among the parties.[15]

On March 28, 2005, City Administrator Noli Aldip sent a letter to the respondents ordering them to vacate the area within the
next thirty (30) days, or be physically evicted from the said property.[16] Respondents sent a letter to the Office of the City
Administrator asserting, in sum, their claim over the subject property and expressing intent for a further dialogue.[17] The
request remained unheeded.

Threatened of being evicted, respondents went to the RTC of Paraaque City on April 21, 2005 and applied for a writ of
preliminary injunction against petitioners.[18] In the course of the proceedings, respondents admitted before the trial court
that they have a pending application for the issuance of a sales patent before the Department of Environment and Natural
Resources (DENR).[19]

On April 29, 2005, the RTC issued an Order[20] denying the petition for lack of merit. The trial court reasoned that respondents
were not able to prove successfully that they have an established right to the property since they have not instituted an action
for confirmation of title and their application for sales patent has not yet been granted. Additionally, they failed to implead the
Republic of the Philippines, which is an indispensable party.

Respondents moved for reconsideration, but the same was denied.[21]

Aggrieved, respondents elevated the matter to the Court of Appeals. On January 31, 2007, the Court of Appeals issued its
Decision in favor of the respondents. According to the Court of Appeals--

The issue ultimately boils down to the question of ownership of the lands adjoining Cutcut Creek particularly Road Lot No. 8
(hereinafter RL 8) and the accreted portion beside RL 8.

The evidentiary records of the instant case, shows that RL 8 containing an area of 291 square meters is owned by Guaranteed
Homes, Inc. covered by TCT No. S-62176. The same RL 8 appears to have been donated by the Guaranteed Homes to the City
Government of Paraaque on 22 March 1966 and which was accepted by the then Mayor FLORENCIO BERNABE on 5 April 1966.
There is no evidence however, when RL 8 has been intended as a road lot.

On the other hand, the evidentiary records reveal that PEDRO VITALEZ possessed the accreted property since 1930 per his
Affidavit dated 21 March 1966 for the purpose of declaring the said property for taxation purposes. The property then became
the subject of Tax Declaration No. 20134 beginning the year 1967 and the real property taxes therefor had been paid for the
years 1966, 1967, 1968, 1969, 1970, 1972, 1973, 1974, 1978, 1980, 1995, 1996, 1997, 1998, 1999, 2000, 2001, 2002, 2003, and
2004. Sometime in 1964 and 1971, construction permits were issued in favor of Appellant MARIO EBIO for the subject property.
On 21 April 1987, PEDRO VITALEZ transferred his rights in the accreted property to MARIO EBIO and his successors-in-interest.

Applying [Article 457 of the Civil Code considering] the foregoing documentary evidence, it could be concluded that Guaranteed
Homes is the owner of the accreted property considering its ownership of the adjoining RL 8 to which the accretion attached.
However, this is without the application of the provisions of the Civil Code on acquisitive prescription which is likewise
applicable in the instant case.

xxxx

The subject of acquisitive prescription in the instant case is the accreted portion which [was] duly proven by the Appellants. It is
clear that since 1930, Appellants together with their predecessor-in-interest, PEDRO VITALEZ[,] have been in exclusive
possession of the subject property and starting 1964 had introduced improvements thereon as evidenced by their construction
permits. Thus, even by extraordinary acquisitive prescription[,] Appellants have acquired ownership of the property in question
since 1930 even if the adjoining RL 8 was subsequently registered in the name of Guaranteed Homes. x x x.

xxxx

Further, it was only in 1978 that Guaranteed Homes was able to have RL 8 registered in its name, which is almost fifty years
from the time PEDRO VITALEZ occupied the adjoining accreted property in 1930. x x x.

xxxx

We likewise note the continuous payment of real property taxes of Appellants which bolster their right over the subject
property. x x x.

xxxx

In sum, We are fully convinced and so hold that the Appellants [have] amply proven their right over the property in question.

WHEREFORE, premises considered, the instant appeal is hereby GRANTED. The challenged Order of the court a quo is
REVERSED and SET ASIDE.

SO ORDERED.[22]
On June 8, 2007, the appellate court denied petitioners motion for reconsideration. Hence, this petition raising the following
assignment of errors:

I. WHETHER OR NOT THE DECISION AND RESOLUTION OF THE HONORABLE COURT OF APPEALS THAT RESPONDENTS HAVE
A RIGHT IN ESSE IS IN ACCORD WITH THE LAW AND ESTABLISHED JURISPRUDENCE[;]

II. WHETHER OR NOT THE DECISION AND RESOLUTION OF THE HONORABLE COURT OF APPEALS THAT THE SUBJECT LOT IS
AVAILABLE FOR ACQUISITIVE PRESCRIPTION IS IN ACCORD WITH THE LAW AND ESTABLISHED JURISPRUDENCE[;] AND

III. WHETHER OR NOT THE STATE IS AN INDISPENSABLE PARTY TO THE COMPLAINT FILED BY RESPONDENTS IN THE LOWER
COURT.[23]

The issues may be narrowed down into two (2): procedurally, whether the State is an indispensable party to respondents action
for prohibitory injunction; and substantively, whether the character of respondents possession and occupation of the subject
property entitles them to avail of the relief of prohibitory injunction.

The petition is without merit.

An action for injunction is brought specifically to restrain or command the performance of an act.[24] It is distinct from the
ancillary remedy of preliminary injunction, which cannot exist except only as part or as an incident to an independent action or
proceeding. Moreover, in an action for injunction, the auxiliary remedy of a preliminary prohibitory or mandatory injunction
may issue.[25]

In the case at bar, respondents filed an action for injunction to prevent the local government of Paraaque City from proceeding
with the construction of an access road that will traverse through a parcel of land which they claim is owned by them by virtue
of acquisitive prescription.

Petitioners, however, argue that since the creek, being a tributary of the river, is classified as part of the public domain, any
land that may have formed along its banks through time should also be considered as part of the public domain. And
respondents should have included the State as it is an indispensable party to the action.

We do not agree.

It is an uncontested fact that the subject land was formed from the alluvial deposits that have gradually settled along the banks
of Cut-cut creek. This being the case, the law that governs ownership over the accreted portion is Article 84 of the Spanish Law
of Waters of 1866, which remains in effect,[26] in relation to Article 457 of the Civil Code.

Article 84 of the Spanish Law of Waters of 1866 specifically covers ownership over alluvial deposits along the banks of a creek. It
reads:

ART. 84. Accretions deposited gradually upon lands contiguous to creeks, streams, rivers, and lakes, by accessions or sediments
from the waters thereof, belong to the owners of such lands.[27]

Interestingly, Article 457 of the Civil Code states:

Art. 457. To the owners of lands adjoining the banks of rivers belong the accretion which they gradually receive from the effects
of the current of the waters.

It is therefore explicit from the foregoing provisions that alluvial deposits along the banks of a creek do not form part of the
public domain as the alluvial property automatically belongs to the owner of the estate to which it may have been added. The
only restriction provided for by law is that the owner of the adjoining property must register the same under the Torrens
system; otherwise, the alluvial property may be subject to acquisition through prescription by third persons.[28]

In contrast, properties of public dominion cannot be acquired by prescription. No matter how long the possession of the
properties has been, there can be no prescription against the State regarding property of public domain.[29] Even a city or
municipality cannot acquire them by prescription as against the State.[30]

Hence, while it is true that a creek is a property of public dominion,[31] the land which is formed by the gradual and
imperceptible accumulation of sediments along its banks does not form part of the public domain by clear provision of law.

Moreover, an indispensable party is one whose interest in the controversy is such that a final decree would necessarily affect
his/her right, so that the court cannot proceed without their presence.[32] In contrast, a necessary party is one whose presence
in the proceedings is necessary to adjudicate the whole controversy but whose interest is separable such that a final decree can
be made in their absence without affecting them.[33]

In the instant case, the action for prohibition seeks to enjoin the city government of Paraaque from proceeding with its
implementation of the road construction project. The State is neither a necessary nor an indispensable party to an action where
no positive act shall be required from it or where no obligation shall be imposed upon it, such as in the case at bar. Neither
would it be an indispensable party if none of its properties shall be divested nor any of its rights infringed.

We also find that the character of possession and ownership by the respondents over the contested land entitles them to the
avails of the action.

A right in esse means a clear and unmistakable right.[34] A party seeking to avail of an injunctive relief must prove that he or
she possesses a right in esse or one that is actual or existing.[35] It should not be contingent, abstract, or future rights, or one
which may never arise.[36]

In the case at bar, respondents assert that their predecessor-in-interest, Pedro Vitalez, had occupied and possessed the subject
lot as early as 1930. In 1964, respondent Mario Ebio secured a permit from the local government of Paraaque for the
construction of their family dwelling on the said lot. In 1966, Pedro executed an affidavit of possession and occupancy allowing
him to declare the property in his name for taxation purposes. Curiously, it was also in 1966 when Guaranteed Homes, Inc., the
registered owner of Road Lot No. 8 (RL 8) which adjoins the land occupied by the respondents, donated RL 8 to the local
government of Paraaque.

From these findings of fact by both the trial court and the Court of Appeals, only one conclusion can be made: that for more
than thirty (30) years, neither Guaranteed Homes, Inc. nor the local government of Paraaque in its corporate or private capacity
sought to register the accreted portion. Undoubtedly, respondents are deemed to have acquired ownership over the subject
property through prescription. Respondents can assert such right despite the fact that they have yet to register their title over
the said lot. It must be remembered that the purpose of land registration is not the acquisition of lands, but only the
registration of title which the applicant already possessed over the land. Registration was never intended as a means of
acquiring ownership.[37] A decree of registration merely confirms, but does not confer, ownership.[38]

Did the filing of a sales patent application by the respondents, which remains pending before the DENR, estop them from filing
an injunction suit?

We answer in the negative.

Confirmation of an imperfect title over a parcel of land may be done either through judicial proceedings or through
administrative process. In the instant case, respondents admitted that they opted to confirm their title over the property
administratively by filing an application for sales patent.

Respondents application for sales patent, however, should not be used to prejudice or derogate what may be deemed as their
vested right over the subject property. The sales patent application should instead be considered as a mere superfluity
particularly since ownership over the land, which they seek to buy from the State, is already vested upon them by virtue of
acquisitive prescription. Moreover, the State does not have any authority to convey a property through the issuance of a grant
or a patent if the land is no longer a public land.[39]

Nemo dat quod dat non habet. No one can give what he does not have. Such principle is equally applicable even against a
sovereign entity that is the State.

WHEREFORE, the petition is DENIED for lack of merit. The January 31, 2007 Decision, as well as the July 8, 2007 Resolution, of
the Court of Appeals in CA-G.R. SP No. 91350 are hereby AFFIRMED.

With costs against petitioners.

Acting Registrar of Land Titles and deeds of Pasay City vs RTC Makati April 26, 1990

SARMIENTO, J.:

The petitioners ** charge His Honor, Judge Francisco Velez, of the Regional Trial Court, Branch 57, Makati, Metro Manila, with
grave abuse of discretion in issuing an order authorizing the private respondent, through Domingo Palomares, to perform acts
of ownership over a 2,574-hectare parcel of land known as Hacienda de Maricabanspread out in various parts of Makati, Pasig,
Taguig, Pasay City, and Parañaque. There is no controversy as to the facts.

On November 5, 1985, the private respondent, Domingo Palomares, as administrator of the heirs of Delfin Casal, commenced
suit with the Regional Trial Court, Branch 132, Makati, Metro Manila for declaratory relief, quieting of title, cancellation of
Transfer Certificate of Title No. 192, and cancellation of entries upon Original Certificate of Title No. 291.

Palomares had earlier come to this Court (February 27, 1985) on a similar petition, and in addition, to direct the Register of
Deeds to issue a duplicate owner's copy of Original Certificate of Title No. 291, embracing allegedly Hacienda de Maricaban, in
lieu of the (alleged) lost one. On September 9, 1985, the Court denied the petition for lack of merit. (G.R. No. 69834).

On December 19, 1985, the petitioners filed their answer.

On June 2, 1986, the private respondent filed a motion to admit amended complaint impleading the Republic of the Philippines
and the Registers of Deeds of Pasig, Makati, and Pasay City as parties-respondents, and alleging, among other things, that: (1)
on October 1, 1906, the Court of Land Registration (James Ostrand, Presiding Judge) confirmed the title of Dolores Pascual Casal
y Ochoa, a native of Madrid, Spain, over the 2,574-hectare parcel above-mentioned; (2) on October 17, 1906, the Register of
Deeds of Rizal issued OCT No. 291 in her name; (3) upon her death, and successive deaths of her heirs, the property devolved
on Gerardo, Reynaldo, Lolita, and Erlinda, all surnamed Casal, great grandchildren of Dolores; (4) no conveyances or
dispositions of any kind have been allegedly made upon the parcel; (5) TCT No. 192, which covers the same landholding, is
allegedly spurious and inexistent; (6) the State itself, by placing 27,213,255 square meters thereof under a military reservation
(Fort McKinley now Fort Bonifacio), by Proclamation No. 423, and fifty hectares thereof pursuant to Proclamation No. 192, had
been guilty of landgrabbing; (7) any and all holders of any and all TCTs emanating therefrom or from TCT No. 192, are null, void,
and of no force and effect; and (8) as a consequence thereof, the heirs of Dolores Casal suffered various damages and
attorney's fees.

On June 26, 1986, the petitioners filed an answer, stating, among other things, that: (1) the estate of Dolores Casal (or Delfin
Casal, her grandchild) is not a juridical person authorized by law to bring suit; (2) the Registers of Deeds of Makati, Pasig, and
Pasay City are not the real parties in interest, but rather, the registered owners over which the court had not acquired
jurisdiction; (3) the non-joinder of the real parties in interest is fatal; (4) OCT No. 291 has long been cancelled; (5) Judge
Gregorio Pineda of the then Court of First Instance of Rizal, Branch XXI, Pasig, had earlier denied prayers for the issuance of
duplicate owner's copy of OCT No. 291 because the land embraced therein had been validly delivered to the Government; (6)
the Supreme Court itself had denied the Casals' appeal; *** (7) as a consequence, res judicata is a bar; (8) prescription has also
set in; and (9) the Casal's claims can not validly override the titles of innocent purchasers for value.
On August 29, 1986, the respondent judge issued a temporary restraining order, directing the petitioners to cease and desist
from performing the acts complained of.

In a subsequent memorandum, the petitioners alleged that Dolores Casal had conveyed the property to the Government of the
United States in 1906 and the Manila Railroad Company on which Judge Ostrand, the Presiding Judge of the Court of Land
Registration, later Justice of this Court, had stamped his imprimatur.

On October 12, 1987, the respondent court issued an order in the tenor, as follows:

No other opposition having been registered, this Court hereby resolves to grant the plaintiffs' prayer in the OMNIBUS MOTION
in order to safeguard the integrity of the land embraced in OCT 291, hereby authorizing for this purpose the plaintiff Domingo
C. Palomares:

1. To order such subdivision and/or individual survey or surveys within Parcel II, Parcel III and Parcel IV under Survey Plan Psu-
2031 by a licensed geodetic engineer or engineers at plaintiffs' expense in order to facilitate and simplify the efficient
administration of the property described in OCT 291; and

2. To sell, exchange, lease or otherwise dispose (of) any area or areas or portion or portions thereof, subject to the approval of
the Intestate Estate Court, to cover expenses for the payment of taxes to which the property is subject, as well as expenses of
administration and for the protection of the integrity of the said lands.

SO ORDERED. 1

Eleven days later, or on October 23, 1987 to be precise, it issued another order, as follows:

Acting on the plaintiffs MOTION dated October 15, 1987 praying for the issuance of a Writ of Execution implementing the Order
of this Court dated October 12, 1987 before the expiration of the time to appeal, and after inquiring from the plaintiff's counsel
for their reason in seeking the same, the Court hereby issues this clarificatory order affirming the power of the plaintiff
Domingo C. Palomares to execute and perform the acts authorized in the said Order of October 12, 1987 without the need of a
Writ of Execution, where no relief has been sought therefrom by any party, said Order being implementable at the instance of
the said plaintiff Domingo C. Palomares, anytime when the said Order becomes final 15 days after the said plaintiff received
copy of the same (see Section 39, Chapter IV, B.P. Blg. 129). Plaintiff Domingo C. Palomares may therefore take whatever steps
he considers appropriate for the implementation of the said Order without need of further Orders or additional authority from
this Court.

SO ORDERED. 2

The petitioners filed a notice of appeal; the respondent court, however, denied it" 3 "it being directed against . . . an
interlocutory order. . . 4

Hence, this recourse.

The petitioners interpose the following questions:

A. Whether or not respondent Court can validly decide before trial in favor of private respondent the ownership and possession
of the 25,743,514 square meters (of) land known as "Hacienda de Maricaban", which is the main issue in this case;

B. Whether or not respondent Court can validly allow private respondent to exercise and perform all acts of ownership and
possession over the said land before trial

C. Whether or not respondent Court has acquired jurisdiction to hear and decide this action;

D. Whether of not respondent Court committed grave abuse of discretion amounting to lack of jurisdiction in not dismissing
this action or allowing petitioners to appeal from the orders in question. 5
In their comment, the private respondent averred, among other things, that: (1) the respondent court, contrary to the
petitioners' claim, did not decide the case "before trial"; (2) OCT No. 291 had not been validly cancelled and that the rubber
stamp impression thereon, "CANCELLED" is a forgery; (3) the act of Judge Pineda, in denying issuance of OCT No. 291, duplicate
owner's copy, can not be considered res judicata because that case involved purportedly a mere petition for issuance of
duplicate owner's copy; (4) non-joinder of proper parties is not a jurisdictional defect; (5) the TCTs issued thereafter are a
nullity because OCT No. 291 had not been shown to have been duly cancelled; (6) OCT No. 291 has become imprescriptible; and
(7) the private respondent has a valid right of dominion over the property.

In the meantime, the private respondent came to this Court on certiorari (G.R. No. 90176) alleging that on December 15, 1987,
in connection with Sp. Proc. No. P-2993 of the Regional Trial Court, Branch 118, Pasay City, entitled "In the matter of the
Intestate Estate of the Late Fortunato Santiago and Mariano Pantanilla Crisanta P. Santiago, et al., Petitioners," Judge Conrado
Vasquez, Jr. issued an order disposing of certain parcels which the private respondent claims as forming part and parcel
of Hacienda de Maricaban.

On June 20, 1988, the respondent judge in G.R. No. 81564 filed his own comment, asserting, among other things, that: (1) what
he had sought to bar, by virtue of injunction, was incursions and forcible entries of trespassers and squatters; (2) the petitioners
can not rightly claim that he had prematurely adjudicated the case, because there was allegedly no decision to begin with; (3)
that he issued the writ of preliminary injunction in order only to maintain the status quo ante bellum that is, to re-place the
private respondent, which had been allegedly in prior possession, in possession; (4) he did not allegedly authorize unbridled
"acts of ownership" to be exercised on the property; (5) all rights of dominion given thereon were subject to the approval of the
intestate estate court; (6) he denied the notice of appeal because the order dated October 12, 1987, was interlocutory in
nature from which no appeal lies; (7) as to jurisdiction, the various motions filed by petitioners, allegedly accepting the court's
jurisdiction, have clothed the court with jurisdiction, and that besides, the jurisdictional question was never raised except now.

On July 7, 1988, the petitioners filed a reply traversing the respondent judge's allegations.

On August 26, 1988, the respondent judge filed a supplemental comment. He reiterated that the writ of injunction was directed
only on such spaces not occupied by the Government (Fort Bonifacio, Libingan ng mga Bayani, Ninoy Aquino International
Airport, Nayong Pilipino, Population Commission, National Science and Development Board, and National Housing Authority).

Meanwhile, Atty. Antonio J. Dalangpan for and on behalf purportedly of the "Heirs of Delfin Casal" and the private respondent,
Domingo Palomares, file a Comment/Opposition in Intervention", dated December 23, 1988 asking for the outright dismissal of
the petition.

On December 14, 1989, the private respondent filed a manifestation, stating, among other things, that assuming OCT No. 291
had been cancelled, there was still basis for the respondent judge to prevent landgrabbers from entering into vacant portions
of the state embraced thereby.

The Court finds the issues, quintessentially, to be:

(1) Is OCT No. 291 still valid and subsisting?

(2) Did the respondent judge, in issuing the orders, dated October 12 and October 23, 1987, commit a grave abuse of discretion
equivalent to lack or excess of jurisdiction?

I.

Is OCT No. 291 still valid and subsisting?

The Court takes judicial notice of the fact that the hectarage embraced by TCT No. 192 (OCT No. 291) consists of Government
property. Three things persuade the Court: (1) the decrees of Proclamations Nos. 192 and 435; (2) the incontrovertible fact that
OCT No. 291 has been duly cancelled; and (3) the division of the Court of Appeals in AC-G.R. CV No. 00293, affirming the
decision of Hon. Gregorio Pineda, Judge of the then Court of First Instance of Rizal, Branch XXI, in LRC (GLRO) Rec. No. 2484,
Case No. R-1467 thereof, entitled "In Re: Issuance of Owner's Duplicate of Certificate of Title No. 291," as well as our own
Resolution, in G.R. No. 69834, entitled "Domingo Palomares, et al., v. Intermediate Appellate Court".

(a)

Proclamation No. 192 ("RESERVING FOR THE VETERANS CENTER SITE PURPOSES CERTAIN PARCEL OF LAND OF THE PUBLIC
DOMAIN SITUATED IN THE PROVINCE OF RIZAL, ISLAND OF LUZON") and Proclamation No. 423 ("RESERVING FOR MILITARY
PURPOSES CERTAIN PARCELS OF THE PUBLIC DOMAIN SITUATED IN THE MUNICIPALITY OF PASIG, TAGUIG, AND PARAÑAQUE
PROVINCE OF RIZAL, AND PASAY CITY") have the character of official assertions of ownership, and the presumption is that they
have been issued by right of sovereignty and in the exercise of the State's dominical authority. We take not only judicial notice
thereof 6 but accept the same as a valid asseveration of regalian light over property.

With respect to the premises occupied by the Libingan ng mga Bayani, Ninoy Aquino International Airport, Nayong Pilipino, the
Population Commission, National Science and Development Board, and the National Housing Authority, we do not have the
slightest doubt that they stand on Government property by sheer presumption that, unless otherwise shown, what the
Government occupies is what the Government owns.

While there is no presumption that property is Government property until otherwise shown, because the law recognizes
private ownership, thus:

Art. 425. Property of private ownership, besides the patrimonial property of the State, provinces, cities, and municipalities,
consists of all property belonging to private persons, either individually or collectively. 7

we find hard evidence on record that: (1) the property covered by OCT No. 291 had been conveyed to the United States of
America; (2) it had been later ceded to the Republic of the Philippines, and (3) as a consequence, OCT No. 291 was cancelled
upon final order of Judge Ostrand.

Be that as it may, the private respondent in G.R. No. 81564 is pressed hard to establish the fact that portions of the property,
especially the open spaces referred to in the lower court's writ of injunction and the private respondent's manifestation of
December 14, 1989, and which open spaces it claims to be outside Maricaban, areindeed outside Maricaban (or OCT 291). With
respect, however, to parts thereof on which Fort Bonifacio, Libingan ng mga Bayani, Ninoy Aquino International Airport,
Nayong Pilipino, Population Commission National Science and Development Board, and National Housing Authority sit, the
hands of the private respondent are tied.

Claims that Judge Ostrand's decree was a counterfeit is not only self-serving, it finds no support from the records. The
presumptions is "that official duty has been regularly performed," 8 and the burden is on the private respondent to prove
irregular performance. The barren insistence that Judge Ostrands order was a forgery is not sufficient to overthrow the
presumption. To begin with, the act of forgery has been seasonably disputed by the petitioners. Secondly, the Acting Registrar
of Deeds of Pasig, who supposedly certified to the fake character of Judge Ostrand's order, has himself joined the other
petitioners in opposing the reconveyance sought.

(b)

The decision in AC-G.R. No. 00293, dismissing the private respondent's petition for the issuance of a new owner's copy of OCT
No. 291, a dismissal affirmed by this Court in G.R. No. 69834, also militates against the return of the property to the heirs of
Delfin Casal. The Appellate Court's judgment, a judgment sustained by this Court, operates as, at the very least, the law of the
case between the parties, that OCT No. 291 has been cancelled and the land covered has been conveyed and ceded to the
National Government. The fact that AC-G.R. CV No. 00293 dealt with a petition for issuance of lost owner's duplicate copy is no
argument because be that as it may, the private respondent can not rightfully say that the heirs of Delfin Casal still have title to
the land. If it can not secure a new owner's copy, it can mean that they have lost title thereto.

(c)
The principle of res judicata is also a bar to the instant proceedings. It should be noted that in G.R. No. 69834, Mr. Domingo
Palomares prayed:

WHEREFORE, premises considered it is most respectfully prayed to the most Honorable Supreme Court, that in the name of
law, justice and fair play, to prevent and frustrate "land-grabbing" by the government, decision be rendered:

FIRST, That a thorough review of the aforementioned resolution of the Intermediate Appellate Court be made;

SECOND, That after due consideration, the resolution subject of review be set aside based on the aforestated assignment of
error;

THIRD, That the Order of the Lower Court dated Jan. 19, 1977 be affirmed as the lawful and valid order;

FOURTH, To erase all doubts by declaring OCT No. 291 as continuously and existing validly against the whole world;

FIFTH, Clearing OCT No. 291 of all adverse claims, since the herein petitioners are the true and legally declared heirs; and

SIXTH, Ordering the Register of Deeds of Pasig, Rizal to issue the Owner's Duplicate Copy of OCT No. 291.

Petitioner-Appellant further prays for other just and equitable reliefs.****

When we therefore denied that petition, we, in effect, held that reconstitution (of lost duplicate owner's copy) was not possible
because the mother title (OCT No. 291) had been duly cancelled. And when we therefore declared OCT No. 291 to have been
cancelled, we perished all doubts as to the invalidity of Mr. Palomares' pretenses of title to Maricaban. Our judgment was
conclusive not only as to Mr. Palomares, but also as to the existing status of the property. As we have held:

The lower Court correctly ruled that the present action is barred by the final judgment rendered in the previous case of Tuason
& Co. vs. Aguila, Civil Case No. Q-4275, of the Court of First Instance of Rizal. The reason is plain: if the herein appellants really
had a preferential right to a conveyance of the land from J.M. Tuason & Co., or if the certificate of (Torrens) title held by Tuason
& Co. were truly void and ineffective, then these facts should have been pleaded by these appellants in the previous case (Q-
4275), since such facts, if true, constituted a defense to the claim of Tuason & Co. for recovery of possession. If appellants failed
to plead such defenses in that previous case, they are barred from litigating the same in any subsequent proceeding, for it is a
well established rule that as between the same parties and on the same subject and cause of action, a final judgment is
conclusive not only on matters directly adjudicated, but also as to any other matter that could have been raised in relation
thereto. 9

II

Did the respondent judge, in issuing the order, dated October 12, 1987, commit a grave abuse of discretion equivalent to lack of
excess of jurisdiction?

(a)

The Court has no doubt that Judge Velez is here guilty of grave abuse of discretion tantamount to lack or excess of jurisdiction
to warrant certiorari. As above-stated, what he gave away, by virtue of reconveyance, was property that inalienably belongs to
the Government or its successors. Worse, he gave away property without notice to the actual possessors, that is, the present
registered owner. It is beyond debate, as we have indicated, that the land had been, since the cancellation of OCT No. 291,
parcelled out to a succession of buyers and owners. In the absence of notice, it acquired no jurisdiction to decree redelivery or
reconveyance. It is well-established that owners of property over which reconveyance is asserted are indispensable parties,
without whom no relief is available and without whom the court can render no valid judgment. 10

Furthermore, the present holders of the land in question are innocent purchasers for value, or presumed to be so in the
absence of contrary evidence, against whom reconveyance does not lie. 11
(b)

The respondent judge can not conceal his faults behind arguments that he did not intend to convey the premises, but rather, to
secure, allegedly, vacant portions thereof from interlopers. First, this is not stated in his order. Second, that order is clear and
unequivocal that Domingo Palomares has the right "[t]o sell, exchange, lease or otherwise dispose of any area or areas or
portion or portions thereof . . . " 12 Third and last, the security of the property is the lookout of the claimants, and not the
court's. In case the premises the respondent judge's injunctive writ have been directed belong to others, let them air their
plaints.

(c)

The Court is also agreed that the challenged order was issued with no benefit of trial or hearing. The private respondent can not
validly rely on AC-G.R. No. 00293 as the "trial or hearing" to justify the issuance of its said order, in the first place, because it is a
different proceeding. But above all, the private respondent itself says that AC-G.R. CV No. 00293 can not be made a basis for
denying reconveyance because "the . . . petition was merely for the issuance of a new owner's duplicate copy . . . 13 Accordingly,
it can not invoke that case and yet, repudiate its effects. It is the height of contradiction.

(d)

It was also grave error for the lower court to deny the Solicitor General's notice of appeal. The Government had all the right to
appeal because: (1) the order of October 12, 1987 was in the nature of a final judgment, as "final judgment" is known in law
(however it is captioned), that is to say, one that "finally disposes of the pending action so that nothing more can be done with
it in the trial court; 14 (2) it did not merely maintain the status quo, but allowed Mr. Domingo Palomares to transact on the
property by near right of dominion over it.

Judge Velez had therefore no reason, indeed, excuse, to deny the Government's notice of appeal. What is plain is the fact that
Judge Velez was hell-bent, so to speak, in blocking the Government's efforts to defend what rightfully belongs to it.

What has obviously been lost on the parties, Judge Velez in particular, is the established principle that injunction does not lie
"to take property out of the possession or control of one party and place it into that of another." 15 In this wise it has also been
held:

xxx xxx xxx

It is a well established doctrine in this jurisdiction that an injunction is not the proper remedy for the recovery of possession of
real estate and the improvements thereon, as well as for the ejectments therefrom of the actual occupants who claim to have
title to or material interest therein. The use of said remedy in such cases has invariably been considered unjustified, in open
violation of the legal presumption that the bona fide possessor of a certain piece of land and improvements thereon, holds the
same under claim of ownership and with a just title, and as an advanced concession of the remedy to which the claimant might
be entitled. (Citations omitted) 16

xxx xxx xxx

Injunction, moreover, is an extraordinary remedy. It lies only in certain cases, to wit:

Sec. 3. Grounds for issuance of preliminary injunction. - A preliminary injunction may be granted at any time after the
commencement of the action and before judgment when it is established:

(a) That the plaintiff is entitled to the relief demanded, and the whole or part of such relief consists in restraining the
commission or continuance of the acts complained of, or in the performance of an act or acts, either for a limited period or
perpetually;

(b) That the commission or continuance of some act complained of during the litigation or the non-performance thereof would
probably work injustice to the plaintiff; or
(c) That the defendant is doing, threatens, or is about to do, or is procuring or suffering to be done, some act probably in
violation of the plaintiffs rights respecting the subject of the action, and tending to render the judgment ineffectual. 17

xxx xxx xxx

The conspicuous and unusual zeal with which Judge Francisco Velez now defends his acts 18 has not escaped us. His Honor
should have borne in mind that in proceedings under Rule 65 of the Rules, such as the present cases, the judge is included only
as a nominal party. Unless otherwise ordained by this Court, he is not called upon to answer or comment on the petition, but
rather, the private respondent. It is indeed distressing to note that it is the very judge who has taken the cudgels for the latter,
in defending its interests, when he, the judge, should have remained a neutral magistrate. Res ipsa loquitor. 19 He must get his
just deserts.

III

The Court thus closes the long-drawn tale of Hacienda de Maricaban. In this connection, let trial judges be cautioned on the
indiscriminate disposition of our dwindling natural resources to private persons. Accordingly, we grant G.R. No. 81564 and
dismiss G.R. No. 90176, and so also, end what has come down as nearly a century of uncertainty, doubt, and conflict Maricaban
has left in its trail. The Court has finally spoken. Let the matter rest.

WHEREFORE:

1. The petition in G.R. No. 81564 is GRANTED:

(a) The Writ of Preliminary Injunction issued by our Resolution, dated April 13, 1988, enjoining the respondent judge from
enforcing his: (i) order of October 12, 1987 and (ii) the follow-up order of October 23, 1987, is made permanent and

(b) Original Certificate of Title No. 291 is declared duly CANCELLED;

2. The petition in G.R. No. 90176 is DISMISSED; and

3. Judge Francisco Velez is ordered to SHOW CAUSE why he should not be administratively dealt with for giving away, by virtue
of reconveyance, property that inalienably belongs to the Government, without notice to the registered owner, and without
benefit of trial or hearing; for blocking Government efforts to defend what rightfully belongs to it; and for filing his comment of
June 17, 1988 and supplemental comment of August 26, 1988 without express leave of court.

Costs against the private respondent.

SO ORDERED.

G.R. No. L-630 November 15, 1947

ALEXANDER A. KRIVENKO, petitioner-appellant,


vs.
THE REGISTER OF DEEDS, CITY OF MANILA, respondent and appellee.

Gibbs, Gibbs, Chuidian and Quasha of petitioner-appellant.


First Assistant Solicitor General Reyes and Solicitor Carreon for respondent-appellee.
Marcelino Lontok appeared as amicus curies.

MORAN, C.J.:

Alenxander A. Kriventor alien, bought a residential lot from the Magdalena Estate, Inc., in December of 1941, the registration of
which was interrupted by the war. In May, 1945, he sought to accomplish said registration but was denied by the register of
deeds of Manila on the ground that, being an alien, he cannot acquire land in this jurisdiction. Krivenko then brought the case
to the fourth branch of the Court of First Instance of Manila by means of a consulta, and that court rendered judgment
sustaining the refusal of the register of deeds, from which Krivenko appealed to this Court.

There is no dispute as to these facts. The real point in issue is whether or not an alien under our Constitution may acquire
residential land.

It is said that the decision of the case on the merits is unnecessary, there being a motion to withdraw the appeal which should
have been granted outright, and reference is made to the ruling laid down by this Court in another case to the effect that a
court should not pass upon a constitutional question if its judgment may be made to rest upon other grounds. There is, we
believe, a confusion of ideas in this reasoning. It cannot be denied that the constitutional question is unavoidable if we choose
to decide this case upon the merits. Our judgment cannot to be made to rest upon other grounds if we have to render any
judgment at all. And we cannot avoid our judgment simply because we have to avoid a constitutional question. We cannot, for
instance, grant the motion withdrawing the appeal only because we wish to evade the constitutional; issue. Whether the
motion should be, or should not be, granted, is a question involving different considerations now to be stated.

According to Rule 52, section 4, of the Rules of Court, it is discretionary upon this Court to grant a withdrawal of appeal after
the briefs have been presented. At the time the motion for withdrawal was filed in this case, not only had the briefs been
prensented, but the case had already been voted and the majority decision was being prepared. The motion for withdrawal
stated no reason whatsoever, and the Solicitor General was agreeable to it. While the motion was pending in this Court, came
the new circular of the Department of Justice, instructing all register of deeds to accept for registration all transfers of
residential lots to aliens. The herein respondent-appellee was naturally one of the registers of deeds to obey the new circular,
as against his own stand in this case which had been maintained by the trial court and firmly defended in this Court by the
Solicitor General. If we grant the withdrawal, the the result would be that petitioner-appellant Alexander A. Krivenko wins his
case, not by a decision of this Court, but by the decision or circular of the Department of Justice, issued while this case was
pending before this Court. Whether or not this is the reason why appellant seeks the withdrawal of his appeal and why the
Solicitor General readily agrees to that withdrawal, is now immaterial. What is material and indeed very important, is whether
or not we should allow interference with the regular and complete exercise by this Court of its constitutional functions, and
whether or not after having held long deliberations and after having reached a clear and positive conviction as to what the
constitutional mandate is, we may still allow our conviction to be silenced, and the constitutional mandate to be ignored or
misconceived, with all the harmful consequences that might be brought upon the national patromony. For it is but natural that
the new circular be taken full advantage of by many, with the circumstance that perhaps the constitutional question may never
come up again before this court, because both vendors and vendees will have no interest but to uphold the validity of their
transactions, and very unlikely will the register of deeds venture to disobey the orders of their superior. Thus, the possibility for
this court to voice its conviction in a future case may be remote, with the result that our indifference of today might signify a
permanent offense to the Constitution.

All thse circumstances were thoroughly considered and weighted by this Court for a number of days and the legal result of the
last vote was a denial of the motion withdrawing the appeal. We are thus confronted, at this stage of the proceedings, with our
duty, the constitutional question becomes unavoidable. We shall then proceed to decide that question.

Article XIII, section 1, of the Constitutional is as follows:

Article XIII. — Conservation and utilization of natural resources.

SECTION 1. All agricultural, timber, and mineral lands of the public domain, water, minerals, coal, petroleum, and other mineral
oils, all forces of potential energy, and other natural resources of the Philippines belong to the State, and their disposition,
exploitation, development, or utilization shall be limited to citizens of the Philippines, or to corporations or associations at least
sixty per centum of the capital of which is owned by such citizens, subject to any existing right, grant, lease, or concession at the
time of the inaguration of the Government established uunder this Constitution. Natural resources, with the exception of public
agricultural land, shall not be alienated, and no licence, concession, or lease for the exploitation, development, or utilization of
any of the natural resources shall be granted for a period exceeding twenty-five years, renewable for another twenty-five years,
except as to water rights for irrigation, water supply, fisheries, or industrial uses other than the development of water "power"
in which cases beneficial use may be the measure and the limit of the grant.

The scope of this constitutional provision, according to its heading and its language, embraces all lands of any kind of the public
domain, its purpose being to establish a permanent and fundamental policy for the conservation and utilization of all natural
resources of the Nation. When, therefore, this provision, with reference to lands of the public domain, makes mention of only
agricultural, timber and mineral lands, it means that all lands of the public domain are classified into said three groups, namely,
agricultural, timber and mineral. And this classification finds corroboration in the circumstance that at the time of the adoption
of the Constitution, that was the basic classification existing in the public laws and judicial decisions in the Philippines, and the
term "public agricultural lands" under said classification had then acquired a technical meaning that was well-known to the
members of the Constitutional Convention who were mostly members of the legal profession.

As early as 1908, in the case of Mapa vs. Insular Government (10 Phil., 175, 182), this Court said that the phrase "agricultural
public lands" as defined in the Act of Congress of July 1, 1902, which phrase is also to be found in several sections of the Public
Land Act (No. 926), means "those public lands acquired from Spain which are neither mineral for timber lands." This definition
has been followed in long line of decisions of this Court. (SeeMontano vs. Insular Government, 12 Phil., 593; Ibañez de
Aldecoa vs. Insular Government, 13 Phil., 159; Ramosvs. Director of Lands, 39 Phil., 175; Jocson vs. Director of Forestry, 39 Phil.,
560; Ankron vs. Government of the Philippines, 40 Phil., 10.) And with respect to residential lands, it has been held that since
they are neither mineral nor timber lands, of necessity they must be classified as agricultural. In Ibañez de Aldecoa vs. Insular
Government (13 Phil., 159, 163), this Court said:

Hence, any parcel of land or building lot is susceptible of cultivation, and may be converted into a field, and planted with all
kinds of vegetation; for this reason, where land is not mining or forestal in its nature, it must necessarily be included within the
classification of agricultural land, not because it is actually used for the purposes of agriculture, but because it was originally
agricultural and may again become so under other circumstances; besides, the Act of Congress contains only three
classification, and makes no special provision with respect to building lots or urban lands that have ceased to be agricultural
land.

In other words, the Court ruled that in determining whether a parcel of land is agricultural, the test is not only whether it is
actually agricultural, but also its susceptibility to cultivation for agricultural purposes. But whatever the test might be, the fact
remains that at the time the Constitution was adopted, lands of the public domain were classified in our laws and jurisprudence
into agricultural, mineral, and timber, and that the term "public agricultural lands" was construed as referring to those lands
that were not timber or mineral, and as including residential lands. It may safely be presumed, therefore, that what the
members of the Constitutional Convention had in mind when they drafted the Constitution was this well-known classification
and its technical meaning then prevailing.

Certain expressions which appear in Constitutions, . . . are obviously technical; and where such words have been in use prior to
the adoption of a Constitution, it is presumed that its framers and the people who ratified it have used such expressions in
accordance with their technical meaning. (11 Am. Jur., sec. 66, p. 683.) Also Calder vs. Bull, 3 Dall. [U.S.], 386; 1 Law. ed., 648;
Bronson vs. Syverson, 88 Wash., 264; 152 P., 1039.)

It is a fundamental rule that, in construing constitutions, terms employed therein shall be given the meaning which had been
put upon them, and which they possessed, at the time of the framing and adoption of the instrument. If a word has acquired a
fixed, technical meaning in legal and constitutional history, it will be presumed to have been employed in that sense in a
written Constitution. (McKinney vs. Barker, 180 Ky., 526; 203 S.W., 303; L.R.A., 1918 E, 581.)

Where words have been long used in a technical sense and have been judicially construed to have a certain meaning, and have
been adopted by the legislature as having a certain meaning prior to a particular statute in which they are used, the rule of
construction requires that the words used in such statute should be construed according to the sense in which they have been
so previously used, although the sense may vary from strict literal meaning of the words. (II Sutherland, Statutory Construction,
p. 758.)
Therefore, the phrase "public agricultural lands" appearing in section 1 of Article XIII of the Constitution must be construed as
including residential lands, and this is in conformity with a legislative interpretation given after the adoption of the Constitution.
Well known is the rule that "where the Legislature has revised a statute after a Constitution has been adopted, such a revision
is to be regarded as a legislative construction that the statute so revised conforms to the Constitution." (59 C.J., 1102.) Soon
after the Constitution was adopted, the National Assembly revised the Public Land Law and passed Commonwealth Act No.
141, and sections 58, 59 and 60 thereof permit the sale of residential lots to Filipino citizens or to associations or corporations
controlled by such citizens, which is equivalent to a solemn declaration that residential lots are considered as agricultural lands,
for, under the Constitution, only agricultural lands may be alienated.

It is true that in section 9 of said Commonwealth Act No. 141, "alienable or disposable public lands" which are the same "public
agriculture lands" under the Constitution, are classified into agricultural, residential, commercial, industrial and for other
puposes. This simply means that the term "public agricultural lands" has both a broad and a particular meaning. Under its broad
or general meaning, as used in the Constitution, it embraces all lands that are neither timber nor mineral. This broad meaning is
particularized in section 9 of Commonwealth Act No. 141 which classifies "public agricultural lands" for purposes of alienation
or disposition, into lands that are stricly agricultural or actually devoted to cultivation for agricultural puposes; lands that are
residential; commercial; industrial; or lands for other purposes. The fact that these lands are made alienable or disposable
under Commonwealth Act No. 141, in favor of Filipino citizens, is a conclusive indication of their character as public agricultural
lands under said statute and under the Constitution.

It must be observed, in this connection that prior to the Constitution, under section 24 of Public Land Act No. 2874, aliens could
acquire public agricultural lands used for industrial or residential puposes, but after the Constitution and under section 23 of
Commonwealth Act No. 141, the right of aliens to acquire such kind of lands is completely stricken out, undoubtedly in
pursuance of the constitutional limitation. And, again, prior to the Constitution, under section 57 of Public Land Act No. 2874,
land of the public domain suitable for residence or industrial purposes could be sold or leased to aliens, but after the
Constitution and under section 60 of Commonwealth Act No. 141, such land may only be leased, but not sold, to aliens, and the
lease granted shall only be valid while the land is used for the purposes referred to. The exclusion of sale in the new Act is
undoubtedly in pursuance of the constitutional limitation, and this again is another legislative construction that the term
"public agricultural land" includes land for residence purposes.

Such legislative interpretation is also in harmony with the interpretation given by the Executive Department of the
Government. Way back in 1939, Secretary of Justice Jose Abad Santos, in answer to a query as to "whether or not the phrase
'public agricultural lands' in section 1 of Article XII (now XIII) of the Constitution may be interpreted to include residential,
commercial, and industrial lands for purposes of their disposition," rendered the following short, sharp and crystal-clear
opinion:

Section 1, Article XII (now XIII) of the Constitution classifies lands of the public domain in the Philippines into agricultural,
timber and mineral. This is the basic classification adopted since the enactment of the Act of Congress of July 1, 1902, known as
the Philippine Bill. At the time of the adoption of the Constitution of the Philippines, the term 'agricultural public lands' and,
therefore, acquired a technical meaning in our public laws. The Supreme Court of the Philippines in the leading case of Mapa
vs. Insular Government, 10 Phil., 175, held that the phrase 'agricultural public lands' means those public lands acquired from
Spain which are neither timber nor mineral lands. This definition has been followed by our Supreme Court in many subsequent
case. . . .

Residential commercial, or industrial lots forming part of the public domain must have to be included in one or more of these
classes. Clearly, they are neither timber nor mineral, of necessity, therefore, they must be classified as agricultural.

Viewed from another angle, it has been held that in determining whether lands are agricultural or not, the character of the land
is the test (Odell vs. Durant, 62 N.W., 524; Lorch vs. Missoula Brick and Tile Co., 123 p.25). In other words, it is the susceptibility
of the land to cultivation for agricultural purposes by ordinary farming methods which determines whether it is agricultural or
not (State vs. Stewart, 190 p. 129).
Furthermore, as said by the Director of Lands, no reason is seen why a piece of land, which may be sold to a person if he is to
devote it to agricultural, cannot be sold to him if he intends to use it as a site for his home.

This opinion is important not alone because it comes from a Secratary of Justice who later became the Chief Justice of this
Court, but also because it was rendered by a member of the cabinet of the late President Quezon who actively participated in
the drafting of the constitutional provision under consideration. (2 Aruego, Framing of the Philippine Constitution, p. 598.) And
the opinion of the Quezon administration was reiterated by the Secretary of Justice under the Osmeña administration, and it
was firmly maintained in this Court by the Solicitor General of both administrations.

It is thus clear that the three great departments of the Government — judicial, legislative and executive — have always
maintained that lands of the public domain are classified into agricultural, mineral and timber, and that agricultural lands
include residential lots.

Under section 1 of Article XIII of the Constitution, "natural resources, with the exception of public agricultural land, shall not be
aliented," and with respect to public agricultural lands, their alienation is limited to Filipino citizens. But this constitutional
purpose conserving agricultural resources in the hands of Filipino citizens may easily be defeated by the Filipino citizens
themselves who may alienate their agricultural lands in favor of aliens. It is partly to prevent this result that section 5 is
included in Article XIII, and it reads as follows:

Sec. 5. Save in cases of hereditary succession, no private agricultural land will be transferred or assigned except to individuals,
corporations, or associations qualified to acquire or hold lands of the public domain in the Philippines.

This constitutional provision closes the only remaining avenue through which agricultural resources may leak into aliens' hands.
It would certainly be futile to prohibit the alienation of public agricultural lands to aliens if, after all, they may be freely so
alienated upon their becoming private agricultural lands in the hands of Filipino citizens. Undoubtedly, as above indicated,
section 5 is intended to insure the policy of nationalization contained in section 1. Both sections must, therefore, be read
together for they have the same purpose and the same subject matter. It must be noticed that the persons against whom the
prohibition is directed in section 5 are the very same persons who under section 1 are disqualified "to acquire or hold lands of
the public domain in the Philippines." And the subject matter of both sections is the same, namely, the non-transferability of
"agricultural land" to aliens. Since "agricultural land" under section 1 includes residential lots, the same technical meaning
should be attached to "agricultural land under section 5. It is a rule of statutory construction that "a word or phrase repeated in
a statute will bear the same meaning throughout the statute, unless a different intention appears." (II Sutherland, Statutory
Construction, p. 758.) The only difference between "agricultural land" under section 5, is that the former is public and the latter
private. But such difference refers to ownership and not to the class of land. The lands are the same in both sections, and, for
the conservation of the national patrimony, what is important is the nature or class of the property regardless of whether it is
owned by the State or by its citizens.

Reference is made to an opinion rendered on September 19, 1941, by the Hon. Teofilo Sison, then Secretary of Justice, to the
effect that residential lands of the public domain may be considered as agricultural lands, whereas residential lands of private
ownership cannot be so considered. No reason whatsoever is given in the opinion for such a distinction, and no valid reason can
be adduced for such a discriminatory view, particularly having in mind that the purpose of the constitutional provision is the
conservation of the national patrimony, and private residential lands are as much an integral part of the national patrimony as
the residential lands of the public domain. Specially is this so where, as indicated above, the prohibition as to the alienable of
public residential lots would become superflous if the same prohibition is not equally applied to private residential lots. Indeed,
the prohibition as to private residential lands will eventually become more important, for time will come when, in view of the
constant disposition of public lands in favor of private individuals, almost all, if not all, the residential lands of the public domain
shall have become private residential lands.

It is maintained that in the first draft of section 5, the words "no land of private ownership" were used and later changed into
"no agricultural land of private ownership," and lastly into "no private agricultural land" and from these changes it is argued
that the word "agricultural" introduced in the second and final drafts was intended to limit the meaning of the word "land" to
land actually used for agricultural purposes. The implication is not accurate. The wording of the first draft was amended for no
other purpose than to clarify concepts and avoid uncertainties. The words "no land" of the first draft, unqualified by the word
"agricultural," may be mistaken to include timber and mineral lands, and since under section 1, this kind of lands can never be
private, the prohibition to transfer the same would be superfluous. Upon the other hand, section 5 had to be drafted in
harmony with section 1 to which it is supplementary, as above indicated. Inasmuch as under section 1, timber and mineral
lands can never be private, and the only lands that may become private are agricultural lands, the words "no land of private
ownership" of the first draft can have no other meaning than "private agricultural land." And thus the change in the final draft is
merely one of words in order to make its subject matter more specific with a view to avoiding the possible confusion of ideas
that could have arisen from the first draft.

If the term "private agricultural lands" is to be construed as not including residential lots or lands not strictly agricultural, the
result would be that "aliens may freely acquire and possess not only residential lots and houses for themselves but entire
subdivisions, and whole towns and cities," and that "they may validly buy and hold in their names lands of any area for building
homes, factories, industrial plants, fisheries, hatcheries, schools, health and vacation resorts, markets, golf courses,
playgrounds, airfields, and a host of other uses and purposes that are not, in appellant's words, strictly agricultural." (Solicitor
General's Brief, p. 6.) That this is obnoxious to the conservative spirit of the Constitution is beyond question.

One of the fundamental principles underlying the provision of Article XIII of the Constitution and which was embodied in the
report of the Committee on Nationalization and Preservation of Lands and other Natural Resources of the Constitutional
Convention, is "that lands, minerals, forests, and other natural resources constitute the exclusive heritage of the Filipino nation.
They should, therefore, be preserved for those under the sovereign authority of that nation and for their posterity." (2 Aruego,
Framing of the Filipino Constitution, p. 595.) Delegate Ledesma, Chairman of the Committee on Agricultural Development of
the Constitutional Convention, in a speech delivered in connection with the national policy on agricultural lands, said: "The
exclusion of aliens from the privilege of acquiring public agricultural lands and of owning real estate is a necessary part of the
Public Land Laws of the Philippines to keep pace with the idea of preserving the Philippines for the Filipinos." (Emphasis ours.)
And, of the same tenor was the speech of Delegate Montilla who said: "With the complete nationalization of our lands and
natural resources it is to be understood that our God-given birthright should be one hundred per cent in Filipino hands . . ..
Lands and natural resources are immovables and as such can be compared to the vital organs of a person's body, the lack of
possession of which may cause instant death or the shortening of life. If we do not completely antionalize these two of our
most important belongings, I am afraid that the time will come when we shall be sorry for the time we were born. Our
independence will be just a mockery, for what kind of independence are we going to have if a part of our country is not in our
hands but in those of foreigners?" (Emphasis ours.) Professor Aruego says that since the opening days of the Constitutional
Convention one of its fixed and dominating objectives was the conservation and nationalization of the natural resources of the
country. (2 Aruego, Framing of the Philippine Constitution, p 592.) This is ratified by the members of the Constitutional
Convention who are now members of this Court, namely, Mr. Justice Perfecto, Mr. Justice Briones, and Mr. Justice Hontiveros.
And, indeed, if under Article XIV, section 8, of the Constitution, an alien may not even operate a small jitney for hire, it is
certainly not hard to understand that neither is he allowed to own a pieace of land.

This constitutional intent is made more patent and is strongly implemented by an act of the National Assembly passed soon
after the Constitution was approved. We are referring again to Commonwealth Act No. 141. Prior to the Constitution, there
were in the Public Land Act No. 2874 sections 120 and 121 which granted aliens the right to acquire private only by way of
reciprocity. Said section reads as follows:

SEC. 120. No land originally acquired in any manner under the provisions of this Act, nor any permanent improvement on such
land, shall be encumbered, alienated, or transferred, except to persons, corporations, associations, or partnerships who may
acquire lands of the public domain under this Act; to corporations organized in the Philippine Islands authorized therefor by
their charters, and, upon express authorization by the Philippine Legislature, to citizens of countries the laws of which grant to
citizens of the Philippine Islands the same right to acquire, hold, lease, encumber, dispose of, or alienate land, or permanent
improvements thereon, or any interest therein, as to their own citizens, only in the manner and to the extent specified in such
laws, and while the same are in force but not thereafter.

SEC. 121. No land originally acquired in any manner under the provisions of the former Public Land Act or of any other Act,
ordinance, royal order, royal decree, or any other provision of law formerly in force in the Philippine Islands with regard to
public lands, terrenos baldios y realengos, or lands of any other denomination that were actually or presumptively of the public
domain or by royal grant or in any other form, nor any permanent improvement on such land, shall be encumbered, alienated,
or conveyed, except to persons, corporations, or associations who may acquire land of the public domain under this Act; to
corporate bodies organized in the Philippine Islands whose charters may authorize them to do so, and, upon express
authorization by the Philippine Legislature, to citizens of the countries the laws of which grant to citizens of the Philippine
Islands the same right to acquire, hold, lease, encumber, dispose of, or alienate land or pemanent improvements thereon or
any interest therein, as to their own citizens, and only in the manner and to the extent specified in such laws, and while the
same are in force, but not thereafter:Provided, however, That this prohibition shall not be applicable to the conveyance or
acquisition by reason of hereditary succession duly acknowledged and legalized by competent courts, nor to lands and
improvements acquired or held for industrial or residence purposes, while used for such purposes:Provided, further, That in the
event of the ownership of the lands and improvements mentioned in this section and in the last preceding section being
transferred by judicial decree to persons,corporations or associations not legally capacitated to acquire the same under the
provisions of this Act, such persons, corporations, or associations shall be obliged to alienate said lands or improvements to
others so capacitated within the precise period of five years, under the penalty of such property reverting to the Government in
the contrary case." (Public Land Act, No. 2874.)

It is to be observed that the pharase "no land" used in these section refers to all private lands, whether strictly agricultural,
residential or otherwise, there being practically no private land which had not been acquired by any of the means provided in
said two sections. Therefore, the prohibition contained in these two provisions was, in effect, that no private land could be
transferred to aliens except "upon express authorization by the Philippine Legislature, to citizens of Philippine Islands the same
right to acquire, hold, lease, encumber, dispose of, or alienate land." In other words, aliens were granted the right to acquire
private land merely by way of reciprocity. Then came the Constitution and Commonwealth Act No. 141 was passed, sections
122 and 123 of which read as follows:

SEC. 122. No land originally acquired in any manner under the provisions of this Act, nor any permanent improvement on such
land, shall be encumbered, alienated, or transferred, except to persons, corporations, associations, or partnerships who may
acquire lands of the public domain under this Act or to corporations organized in the Philippines authorized thereof by their
charters.

SEC. 123. No land originally acquired in any manner under the provisions of any previous Act, ordinance, royal order, royal
decree, or any other provision of law formerly in force in the Philippines with regard to public lands terrenos baldios y
realengos, or lands of any other denomination that were actually or presumptively of the public domain, or by royal grant or in
any other form, nor any permanent improvement on such land, shall be encumbered, alienated, or conveyed, except to
persons, corporations or associations who may acquire land of the public domain under this Act or to corporate bodies
organized in the Philippines whose charters authorize them to do so: Provided, however, That this prohibition shall not be
applicable to the conveyance or acquisition by reason of hereditary succession duly acknowledged and legalized by competent
courts: Provided, further, That in the event of the ownership of the lands and improvements mentioned in this section and in
the last preceding section being transferred by judicial decree to persons, corporations or associations not legally capacitated to
acquire the same under the provisions of this Act, such persons, corporations, or associations shall be obliged to alienate said
lands or improvements to others so capacitated within the precise period of five years; otherwise, such property shall revert to
the Government.

These two sections are almost literally the same as sections 120 and 121 of Act No. 2874, the only difference being that in the
new provisions, the right to reciprocity granted to aliens is completely stricken out. This, undoubtedly, is to conform to the
absolute policy contained in section 5 of Article XIII of the Constitution which, in prohibiting the alienation of private
agricultural lands to aliens, grants them no right of reciprocity. This legislative construction carries exceptional weight, for
prominent members of the National Assembly who approved the new Act had been members of the Constitutional Convention.

It is said that the lot question does not come within the purview of sections 122 and 123 of Commonwealth Act No. 141, there
being no proof that the same had been acquired by one of the means provided in said provisions. We are not, however, diciding
the instant case under the provisions of the Public Land Act, which have to refer to land that had been formerly of the public
domain, otherwise their constitutionality may be doubtful. We are deciding the instant case under section 5 of Article XIII of the
Constitution which is more comprehensive and more absolute in the sense that it prohibits the transfer to alien of any private
agricultural land including residential land whatever its origin might have been.

And, finally, on June 14, 1947, the Congress approved Republic Act No. 133 which allows mortgage of "private real property" of
any kind in favor of aliens but with a qualification consisting of expressly prohibiting aliens to bid or take part in any sale of such
real property as a consequence of the mortgage. This prohibition makes no distinction between private lands that are strictly
agricultural and private lands that are residental or commercial. The prohibition embraces the sale of private lands of any kind
in favor of aliens, which is again a clear implementation and a legislative interpretation of the constitutional prohibition. Had
the Congress been of opinion that private residential lands may be sold to aliens under the Constitution, no legislative measure
would have been found necessary to authorize mortgage which would have been deemed also permissible under the
Constitution. But clearly it was the opinion of the Congress that such sale is forbidden by the Constitution and it was such
opinion that prompted the legislative measure intended to clarify that mortgage is not within the constitutional prohibition.

It is well to note at this juncture that in the present case we have no choice. We are construing the Constitution as it is and not
as we may desire it to be. Perhaps the effect of our construction is to preclude aliens, admitted freely into the Philippines from
owning sites where they may build their homes. But if this is the solemn mandate of the Constitution, we will not attempt to
compromise it even in the name of amity or equity. We are satisfied, however, that aliens are not completely excluded by the
Constitution from the use of lands for residential purposes. Since their residence in the Philippines is temporary, they may be
granted temporary rights such as a lease contract which is not forbidden by the Constitution. Should they desire to remain here
forever and share our fortunes and misfortunes, Filipino citizenship is not impossible to acquire.

For all the foregoing, we hold that under the Constitution aliens may not acquire private or public agricultural lands, including
residential lands, and, accordingly, judgment is affirmed, without costs.

Feria, Pablo, Perfecto, Hilado, and Briones, JJ., concur.

Collado vs CA Oct. 4, 2002

G.R. No. L-33676 June 30, 1971

MARIANO PAJOMAYO, PATRICIO PAJOMAYO, EUSEBIO PAJOMAYO, SOLEDAD PAJOMAYO assisted by her husband
FLORIANO CHITONGCO, DEMFTRIO PAJOMAYO, CRISTITA PAJOMAYO assisted by her husband MANUEL RAMIREZ,
PATROCINIO PAJOMAYO and CRISPO PAJOMAYO, plaintiffs-appellees,
vs.
RODRIGO MANIPON and PERFECTA ZULUETA, defendants-appellants.

ZALDIVAR, J.:

Appeal from the decision of the Court of First Instance of Pangasinan (Branch IX) in its Civil Case No. U-655. The decision was
originally appealed to the Court of Appeals on November 3, 1964. In the resolution of the second special division of the Court of
Appeals, promulgate on April 27, 1971, this case was certified to this Court as one that is within the exclusive appellate
jurisdiction of the Supreme Court — only errors on question of law being involved in the appeal. 1

On June 5, 1963 the plaintiffs filed in the Court of First Instance of Pangasinan (Branch IX, at Urdaneta, Pangasinan) a complaint
alleging that they are owners pro-indiviso of the parcel of land described in the complaint which is covered by Original
Certificate of Title No. 1089 in the name of Diego Pajomayo, issued by the office of the Register of Deeds of Pangasinan; that
they had acquired the land as an inheritance from their late father Diego Pajomayo; that they and their predecessor-in-interest
had been in actual, peaceful and uninterrupted possession of said property in the concept of owners for a period of more than
70 years until the early part of the year 1956 when the defendants dispossessed them of said property, resulting in their having
suffered annual damages amounting to around P1,100.00 representing the value of the crops of rice; mongo, corn and
vegetables that they failed to harvest; and that because they have to file the present suit they must spend P800.00 for
incidental expenses of litigation and attorney's fees. The plaintiffs prayed that they be declared the lawful owners pro-indiviso
of the land in question, and that the defendants be ordered to vacate the land and pay them the damages they have suffered.

In their answer the defendants, after denying some of the allegations of the complaint, alleged that they are the exclusive
owners of a parcel of land covered by Original Certificate of Title No. 14043 issued by the office of the Register of Deeds of
Pangasinan, the said land having been adjudicated to them in the cadastral proceedings of the Malasique cadastre and that
apparently the plaintiffs are claiming the same parcel of land. The defendants claim they had acquired the land mentioned in
their answer by inheritance from their deceased father Pioquinto Manipon, and that they and their predecessors-in-interest
have been in actual, peaceful, and adverse possession of said land for more than 70 years, to the reclusion of plaintiffs; and that
as possessors in good faith they have introduced on the land improvements worth P1,000.00. As affirmative defenses, the
defendants allege that plaintiffs' action is barred by res-judicata and/or prescription and that the court has no jurisdiction over
the subject matter of the case. The defendants set up a counterclaim for damages in the sum of P500.00 representing
attorney's fees that they paid their counsel. The defendants prayed that they be declared the owners of the parcel of land
mentioned in their answer; that the plaintiffs be ordered to pay them damages in the sum of P500.00; and, in the alternative
should judgment be rendered against them, that the plaintiffs ordered jointly and severally to pay them the sum of P1,000.00
representing the value of the improvements they have introduced on the land.

When the case, was called for trial on July 6, 1964, the counsels for the parties submitted to the court a stipulation of facts, as
follows:

1. That plaintiffs are the children and compulsory heirs of the late Diego Pajomayo;

2. That parties agree that the land in question is covered by two Certificates of Title, one in the name of Diego Pajomayo under
Original Cert. of Title No. 1089 issued under Free Patent, owner's copy attached hereto as Annex A; and Original Cert. of Title
No. 14034, in the name of the Defendant Rodrigo Manipon, issued in Cadastral Case No. 91 of Malasique Cadastre, certified
true copy of which is attached hereto as Annex B;

3. That parties agree to submit this case on the above stipulations without further presentation of evidence.

WHEREFORE, it is respectfully prayed this Honorable Court that decision be rendered upon the foregoing stipulation after the
parties have submitted simultaneous memoranda within a period of twenty (20) days from today.

Urdaneta, Pangasinan this 6th day of July, 1964.

On the basis of the foregoing stipulation of facts, the Court of First Instance of Pangasinan (Branch IX) made a finding that
Original Certificate of Title No. 1089 held by the plaintiffs was issued earlier than Original Certificate of Title No. 14034 held by
the defendants, and on September 10, 1964 it rendered a decision, the dispositive portion of which reads as follows:

WHEREFORE, the Court, rendering judgment in favor of the plaintiffs and against the defendants, hereby orders the latter to
vacate the land in question and deliver possession thereof to the former who are entitled thereto as the heirs of Diego
Pajomayo who is hereby declared the legal and lawful owner of the said property.

The Register of Deeds for Pangasinan is hereby ordered to cancel de oficio Original Certificate of Title No. 14034.

With costs of this suit against the defendant.

From the above-mentioned decision of the lower court, the defendants brought up the present appeal. In their appeal the
defendants made the following assignment of errors:

1. The lower court erred in declaring Original Certificate of Title No. 14034 of herein appellants null and void notwithstanding
the fact that this is not one of the reliefs prayed for by the appellees.

2. The lower erred in ordering the herein appellants to vacate the land in question and to deliver the possession thereof to the
herein appellees although the latter failed to prove their cause of action against the herein appellants.
3. The lower court erred in not applying the doctrine of res judicata in favor of herein appellants.

The appeal has no merit. There is no question regarding the identity of the land involved. The only question to be resolved in
the present appeal is: which of the two original certificates of title should prevail — the No. 1089 held by the plaintiffs-
appellees which was issued in virtue of the homestead patent, or the No. 14034 held by the defendants-appellants which was
issued in connection with the cadastral proceedings? Necessarily when one of the two titles is held to be superior over the
other, one should be declared null and void and should be ordered cancelled. And if a party is declared to be the owner of a
parcel of land pursuant to a valid certificate of title said party is entitled to the possession of the land covered by said valid title.
The decree of registration issued in the cadastral proceedings does not have the effect of annulling the title that had previously
been issued in accordance with the provisions of the land Registration Law (Act 496).

The lower court, therefore, had correctly ordered the cancellation of Certificate of Title No 14034 held by the defendants when
it declared that Original Certificate of Title No. 1089 held by the plaintiffs should prevail. Likewise, the lower court had correctly
ordered the defendants to vacate the land in question and deliver possession thereof to plaintiffs after declaring plaintiffs
entitled thereto as the heirs of Diego Pajomayo, the lawful owner of the land.

Contrary to the claim of defendants, the doctrine of res judicata can not be applied in their favor in the present case.

The undisputed fact is that the plaintiffs base their claim of title to the land in question on Original Certificate of Title No. 1089
issued to their father, Diego Pajomayo, on November 27, 1931 in virtue of a free patent that was granted to him. The law
requires that the homestead patent must be registered in the office of the Register of Deeds of the province where the land
covered by the patent lies. Section 122 of the Land Registration Act (Act 496) provides as follows:

SEC. 122. Whenever public lands in the Philippine Islands belonging (to the Government of the United States or) to the
Government of the Philippine Islands are alienated, granted, or conveyed to persons or to public or private corporations, the
same shall be brought forthwith under the operation of this Act and shall become registered lands. It shall be the duty of the
official issuing the instrument of alienation, grant, or conveyance in behalf of the Government to came suck instrument before
its delivery to the grantee, to be filed with the register of deeds for the province where the land lies and to be there registered
like other deeds and conveyance, whereupon a certificate shall be entered as in other cases of registered land and an owner's
duplicate certificate issued to the grantee. The deed, grant, or instrument of conveyance from the Government to the grantee
shall not take effect as a conveyance or bind the land, but shall operate only as contract between the Government and the
grantee and as evidence of authority to the clerk or register of deeds to make registration. The act of registration shall be the
operative act to convey and effect the land, and in all cases under this Act registration shall be made in the office of the register
of deeds for the province where the land lies. The fees for registration shall be paid by the grantee. After due registration and
issue of the certificate and owner's duplicate, such land shall be registered land for all purposes under this Act. (Emphasis
supplied).

Thus, it has been ruled by this Court that once a homestead patent granted in accordance with the Public Land Act registered
pursuant to Section 122 of Act 496 (Land Registration Act), the certificate of title issued in virtue of said patent has the force
and effect of a Torrens Title under the Land Registration Act. In the cage of Aquino vs. Director of Lands, 39 Phil. 850, this Court
held:

The procedure under the Land Registration Law and under the provisions of Chapter VI of the Public Land Law are the same in
that both are against the whole world, both take the nature of judicial proceedings, and for both the decree of registration
issued is conclusive and final. (Act No. 496, secs. 35, 38, and 45, as amended; Act 926. secs. 59 and 63, as amended; Escueta vs.
Director of Lands, 16 Phil. 482; Grey Alba vs. De la Cruz, 17 Phil. 49; Roxas vs. Enriquez, 29 Phil. 31; Legarda, et al. vs. Saleeby,
31 Phil. 591) ... 2

In the case of Manalo vs. Lukban and Liwanag, 48 Phil. 973, 979, this Court said:

The record shows that the land covered by said judgment had already been granted by the government to Monico Corpus
Manuel as homesteader under the provisions of Act 926, the corresponding certificate of title having been registered and
issued to said grantee. By virtue of said registration and issuance of the certificate of title, that land is considered registered
within the meaning of the Land Registration Act, No. 496 (sec. 122 of said Act).

So that when the trial was held in the cadastral proceeding which covered said land, and when the judgment of June 29, 1922
concerning said land was rendered in said proceeding, the title to that land could no longer be the subject of any inquiry,
determination or judgment, for it had already been adjudicated to Monico Corpus Manuel more than ten years before, with all
the legal formalities and with all the force of a title under Act 496.

The doctrine laid down in the two cases above-cited has been affirmed and applied by this Court in a long line of
decisions. 3 The ruling regarding the validity and force of a certificate of title issued in virtue of the registration of a homestead
patent is applicable to certificates of title issued in virtue of the registration of other land patents under the Public land Law. In
the case of Lahora, et al. vs. Dayanghirang, et al., G.R. No. L-28565, January 30, 1971, 4 thus Court, speaking through Mr.
Justice J.B.L. Reyes, held:

The rule in this jurisdiction, regarding public land patents and the character of the certificate of title that may be issued by
virtue thereof, is that where land is granted by the government to a private individual, the corresponding patent therefor is
recorded, and the certificate of title is issued to the grantee; thereafter, the land is automatically brought within the operation
of the Land Registration Act, the title issued to the grantee becoming entitled to all the safeguards provided in Section 38 of
said Act. In other words, upon the expiration of one year from its issuance, the certificate of title becomes irrevocable and
indefeasible like a certificate issued in a registration proceeding.

It is the settled rule in this jurisdiction that where two certificates of title are issued to different persons covering the same land
in whole or in part, the earlier in date must prevail as between the original parties, and in case of successive registration where,
more than one certificate is issued over the land the person holding under the prior certificate is entitled to the land as against
the person who relies on the second certificate. 5

In the case now before Us, it appearing that Original Certificate of Title No. 14034 upon which the defendant appellants base
their claim of ownership over the land in question was issued on April 1, 1957, while Original Certificate of Title No. 1089 upon
which plaintiffs-appellees base a similar claim was issued on November 27, 1931, under the law and the authorities. We have
herein cited, the latter certificate of title should prevail, and the former should be cancelled.

WHEREFORE, the decision appealed from should be, as it is hereby, affirmed, with costs against the defendants-appellants. It is
so ordered.

Concepcion, C.J., Reyes, J.B.L Dizon, Makalintal, Fernando, Teehankee, Barredo, Villamor and Makasiar, JJ., concur.

Castro, J., is on leave.

You might also like